FOR DOWNLOADING BOOKS : Page-1

42
TARGET NO-1 CLASSES (A UNIT OF) EXECUTIVE MAKERS PVT.LTD. V-157 FOR DOWNLOADING BOOKS : www.executivemakers.com Page-1

Transcript of FOR DOWNLOADING BOOKS : Page-1

TARGET NO-1 CLASSES (A UNIT OF) EXECUTIVE MAKERS PVT.LTD. V-157

FOR DOWNLOADING BOOKS : www.executivemakers.com Page-1

TARGET NO-1 CLASSES (A UNIT OF) EXECUTIVE MAKERS PVT.LTD. V-157

FOR DOWNLOADING BOOKS : www.executivemakers.com Page-2

Book code-V157 For online books

www.executivemakers.com

FCI SOLVED SET PAPERS

(IN ENGLISH) Published by:

Executive Makers Private Limited Corporate office:

Sai plaza {Basement} Sardhana Road, Kanker khera

Meerut cantt., UP-250001

Email-(1) [email protected] Email-(2) [email protected]

Contact us : 8445003800 , 8445004018

For Downloading Books

www.executivemakers.com

M.D- MR. VIKAS TOMAR

BE TARGETIANS & GET THE PERFECTION

BE EXECUTIVE WITH EXECUTIVE MAKERS

Batches are for 3, 4 , 6, 12 & 18 Months.

SELECTIONS A UNIQUE PLACE

FOR SSC, BANKING, RAILWAY, POLICE, NDA, CDS, ARMY, NAVY, AIRFORCE, IAS, PCS, TET, CTET,

TGT, PGT, BED, NET, CAT, MAT, CSAT, CLAT, AFCAT AND OTHER GENERAL COMPETITIONS

WITH

INTERVIEW PREPARATION

APPOINTMENT

SHORT TRICKS MATHS

& REASONING

Fresh Batches start from 1st , 11th & 21st of every

month. Like us at facebook @ 123Readyforjob

HELPLINE NO. FOR CLASSES…8445003800

*

*

*

JOIN ONE YEAR SPECIAL CLASS PROGRAMME

AND GET SELECTED IN THE FUTURE EXAMS

TARGET NO-1 CLASSES (A UNIT OF) EXECUTIVE MAKERS PVT.LTD. V-157

FOR DOWNLOADING BOOKS : www.executivemakers.com Page-3

PRACTICE SET-1 General Intelligence & Reasoning

Directions (1-5) : In each of the following questions, select the related letters/word/number from the given alternatives. 1. SKIT : TIKS : : ? : FLAP

(1) PLFA (2) PFAL (3) PLAF (4) PALF

2. 6 : 36 : : 9 : ? (1) 81 (2) 98 (3) 42 (4) 56

3. 384 : ? : : 216 : 63 (1) 128 (2) 124 (3) 113 (4) 192

4. Asthma : Lungs : : Conjunctivitis : ? (1) Bones (2) Teeth (3) Eyes (4) Blood

5. Thermometer : Temperature : : Glucometer : ? (1) Body Sugar (2) Body Resistance (3) Blood (4) Blood Sugar

Directions (6-8) : In each of the following questions, find the odd word/number/letters from the given alternatives. 6. (1) 4249 (2) 7586

(3) 4623 (4) 9781 7. (1) X (2) P

(3) D (4) I 8. (1) Reed (2) Pit

(3) Tab (4) Hold Directions (9-14) : In each of the following questions, a series is given, with one term missing. Choose the correct alternative from the given one that will complete the series. 9. ZA, VC, RE, NG, ?, FK

(1) OH (2) KI (3) IJ (4) JI

10. 4, 16, 36, ?, 100 (1) 56 (2) 64 (3) 48 (4) 52

11. 12, 23, 34, 45, ?, 67, 78, 89 (1) 76 (2) 69 (3) 56 (4) 54

12. ACE, BDF, GIK, ? (1) JHM (2) HJI (3) HJL (4) MJH

13. DEF, HIJ, MNO, ? (1) RTV (2) SRQ (3) RST (4) STU

14. If 20 + 15 = 24 and 64 + 13 = 42, then 11 + 28 = ? (1) 33 (2) 36 (3) 31 (4) 34

15. Reena walks 4 km East and then cycles 10 km North, turns to the left and covers 4 km and once again turns to the North and moves 2 km. How far is she from the starting point?

(1) 10 km (2) 14 km (3) 8 km (4) 12 km

16. In a certain code, ‘CAPITAL’ is written as ‘CPATILA’, How is ‘PERSONS’ written in that code?

(1) PSONRES (2) PONSRES

(3) PESONRS (4) PREOSSN 17. in the following problem :

= stands for + stands for –

stands for = - stands for > > stands for +

< stands for

stands for < When these new symbols are substituted, only one will be correct. Identify the correct equation.

(1) 24 = 4 > 5 17 + 13

(2) 24 > 4 + 5 17 + 13

(3) 24 = 4 > 5 17 > 13 (4) None of the above

Directions (18-20) : In each of the following questions, select the missing number from the given responses. 18. 5 7 4 39

6 9 5 59 7 11 ? 83

(1) 6 (2) 7 (3) 4 (4) 8

19. 8 16 10 2 8 5 4 ? 9 8 5 11

(1) 2 (2) 5 (3) 4 (4) 3

20. 8 (14) 15 13 (18) 22 25 (?) 41 (1) 33 (2) 42 (3) 14 (4) 32

21. Which of the answer figures is exactly the mirror-image of the given figure, when the mirror is held as shown? Question Figure :

Answer Figure :

(1) (2)

(3) (4) 22. A triangular sheet of paper has been folded and punched

as shown in the figure. You have to figure out from amongst the four response figures, how it will appear when opened? Question Figures :

Answer Figures :

TARGET NO-1 CLASSES (A UNIT OF) EXECUTIVE MAKERS PVT.LTD. V-157

FOR DOWNLOADING BOOKS : www.executivemakers.com Page-4

(1) (2)

(3) (4) 23. Which figure represents the relationship among Hockey

Players, Rubber Balls, Poets?

(1) (2)

(3) (4)

(5) (1) 1 (2) 4 (3) 2 (4) 5

24. How many squares are there is a given figure?

(1) 12 (2) 13 (3) 10 (4) 11

Directions (25) : In the following question, two statements ate followed by two conclusions I and II. You have to consider the statements to be true, even if they seem to be at variance from commonly known facts. You have to decide which of the given conclusions if any, definitely follows from the given statements. 25. Statements :

I. Parental role in the education of their children is very significant. II. All parents must realise this Conclusions : I. Parentless children cannot develop to their maximum potential. II. Parents can help their children in their education.

(1) Neither conclusion I nor II follows (2) Both conclusions I and II follow (3) Only conclusion I follows (4) Only conclusion II follows

General Awareness 26. There is no selling cost under

(1) oligopoly (2) duopoly (3) perfect competition (4) monopolistic competition

27. Which of the following systems in independent India goes against the very basis of democracy?

(1) Party system (2) Parliamentary system (3) Caste system (4) Economic system

28. Which one of the following statements is correct? (1) The value of a commodity depends upon its price. (2) The value of a commodity is entirely dependent upon the substitutes.

(3) A commodity will have value if it is wanted by somebody. (4) A commodity will have value only if it is scarce relative to demand.

29. For controlling inflation, the central bank should (1) purchase Government securities in the open market (2) lower the reserve ratio of the banks (3) sell Government securities in the open market (4) lower the bank rate

30. Match the medieval travellers with their countries.

A. Marco Polo 1. Spain

B. Ibn Batuta 2. Balkh

C. Antonio Monserrate 3. Italy

D. Mahmud Wali Balkhi 4. Morocco

A B C D (1) 3 4 1 2 (2) 1 3 2 4 (3) 4 3 1 2 (4) 3 1 4 2

31. Rana Kumbha constructed the Tower of Victory at Chittor to mark his victory over

(1) The Khan of Nagaur (2) Roa Jodha of Marwar (3) Ahmad Shah of Gujarat (4) Mahmud Khalji of Malwa

32. Match the following:

A. Sarojini Naidu 1. Muslim League

B. M.A. Jinnah 2. Indian National

Congress

C. Sir Tej Bahadur

Sapru

3. Hindu

Mahasabha

D. V.D. Savarkar 4. Liberal Party

A B C D (1) 2 4 1 3 (2) 4 1 3 2 (3) 2 1 4 3 (4) 2 1 3 4

33. A citizen can directly move the Supreme Court for any violation of Fundamental Rights under

(1) Article-33 (2) Article-34 (3) Article-31 (4) Article-32

34. Which one of the following sitting Vice-Presidents of India contested for the post of President and lost the election?

(1) S. Radhakrishnan (2) V.V. Giri (3) Bhairon Singh Shekhawat (4) Both (2) and (3)

35. Identify the medical trio of ancient India from the following names

(1) Charaka, Susruta and Bharata (2) Charaka, susruta and Patanjali

TARGET NO-1 CLASSES (A UNIT OF) EXECUTIVE MAKERS PVT.LTD. V-157

FOR DOWNLOADING BOOKS : www.executivemakers.com Page-5

(3) Charaka, Susruta and Vagbhata (4) Charaka, Vatsyayana and Vagbhata

36. Vegetation is effective in absorbing (1) high frequency sound (2) pollutant metals (3) pollutant gases (4) polluted water

37. The fundamental role of root hairs in plants is (1) to absorb water and mineral salts from the soil . (2) to bind soil particles to the root for firm fixation of the plant. (3) to protect the young root from damage by coarse soil parties. (4) to protect the root from soil microbes.

38. Which one of the following acts as a mediator between the user’s program and the hardware?

(1) Operating System (2) Browser (3) Compiler (4) Editor

39. Petrology is the study of (1) earth (2) minerals (3) rocks (4) soils

40. The red colour of ripe tomatoes is due to the presence of (1) chlorophyll (2) carotenoids (3) hormones (4) vitamins

41. Anticoagulants are not present in (1) leech (2) wasp (3) mosquito (4) bed bug

42. The substances which have infinite electrical resistance are called

(1) conductors (2) resistors (3) insulators (4) condensers

43. A bioenergy source obtained by fermentation to supplement fossil fuel petrol is

(1) Diesel (2) Methane (3) Kerosene (4) Ethanol

44. A liquid is said to boil when its (1) vapour pressure is greater than the surrounding pressure. (2) vapour pressure is less than the surrounding pressure. (3) vapour pressure equals the surrounding pressure. (4) vapour pressure vanishes to zero.

45. PVC is obtained by the polymerisation of (1) Styrene (2) Acetylene (3) Propene (4) Vinyl chloride

46. Stainless steel is an alloy of (1) iron, carbon and zinc (2) iron, zinc and maganese (3) iron, chromium and nickel (4) iron, chromium and carbon

47. Which one of the following iron and steel works in India is not under public sector?

(1) Bhilai (2) Durgapur (3) Bokaro (4) Jamshedpur

48. The first woman to conquer Mount Everest twice, is (1) Santosh Yadav (2) Suma Shirur (3) Suraj Lata Devi (4) Jyoti Randhawa

49. What is the name of the first research station established by India in the Antarctica?

(1) Agnihotri (2) Aryabhatta

(3) Dakshin Gangotri (4) Maitri 50. Which of the following is not correctly paired?

(1) Harbhajan Singh - Kabaddi (2) Saina Nehwal - Badminton (3) Jwala Gutta - Tennis (4) Virat Kohli – Cricket

Quantitative Aptitude 51. A bookseller makes 8% profit after selling the book at

10% discount. The ratio of the cost price to the marked price is

(1) 5 : 6 (2) 6 : 5 (3) 4 : 5 (4) 5 : 4

52. A certain sum of money is distributed to A and B in the ratio 2 : 5. If A received Rs. 100, then the money received by B is

(1) Rs. 250 (2) Rs. 300 (3) Rs. 200 (4) Rs. 150

53. What is the value of (2.1)2 × 0.0441 = ? (1) 92.51 (2) 0.9251 (3) 0.9261 (4) 92.61

54. The greatest number that can divide 140, 176, 264 leaving remainders of 4, 6 and 9 respectively is

(1) 17 (2) 2 (3) 85 (4) 34

55. If 10 men or 18 boys can do a work in 15 days, then the number of days required by 15 men and 33 boys to do twice the work is

(1) 9 (2) 36

(3) 4 1

2 (4) 8

56. The value of 13723

× 14583

is (1) 106 (2) 136 (3) 116 (4) 126

57. The base of a triangle is 2 cm more than twice its altitude. If the area is 12 sq. cm, its altitude will be

(1) 4 cm (2) 3 cm (3) 6 cm (4) 5 cm

58. A man leaves Rs. 12,600 to be divided among 7 sons, 3 daughters and 5 nephews. If each daughter receives three times as much as each nephew and each son seven times as much as each nephew, then each daughter’s share is

(1) Rs. 600 (2) Rs. 750 (3) Rs. 700 (4) Rs. 650

59. By selling an article for Rs. 21,000 a man gains 5%. To get a profit of 15%, he has to sell it for

(1) Rs. 23,000 (2) Rs. 25,000 (3) Rs. 19,800 (4) Rs. 20,700

60. The salary of an employee increases every year in the month of July by 10%. If his salary in May 2000 was Rs. 15,000 .his salary in October 2001 was

(1) Rs. 18,150 (2) Rs. 19,965 (3) Rs. 16,500 (4) Rs. 18,000

61. If the volume and the surface area of a sphere are numerically equal, then the numerical value of the radius of the sphere is

(1) 3 (2) 4 (3) 1 (4) 2

TARGET NO-1 CLASSES (A UNIT OF) EXECUTIVE MAKERS PVT.LTD. V-157

FOR DOWNLOADING BOOKS : www.executivemakers.com Page-6

62. Minimum value of 𝑥2 +1

𝑥2+1− 3 is

(1) 0 (2) -1 (3) -3 (4) -2

63. If (3x – y) : (x + 5y) = 5 : 7, then the value of (x + y) : (x – y) is

(1) 2 : 3 (2) 3 : 2 (3) 3 : 1 (4) 1 : 3

64. In ABC, B = 600, C = 40

0. If AD bisects BAC and AE

BC, then EAD is (1) 10

0 (2) 20

0

(3) 400 (4) 80

0

65. If the numerical value of the volume of a right circular cylinder and its curved surface area are equal, then its radius is

(1) 3 units (2) 6 units (3) 2 units (4) 4 units

66. If in ABC, A = 900, BC = a, AC = b and AB = c, then the

value of tan B + tan C is

(1) 𝑐2

𝑎𝑏 (2)

𝑎2+𝑐2

𝑏

(3) 𝑏2

𝑎𝑐 (4)

𝑎2

𝑏𝑐

67. tan 70 tan 23

0 tan 60

0 tan 67

0 tan 83

0 is equal to

(1) 0 (2) 3

(3) 1

3 (4) 1

68. The value of (sec - cos ) (cosec - sin ) (tan + cot ) is

(1) 1 (2) 3

2

(3) 2 (4) 0

69. G is the centroid of ABC. If AG = BG, then BGC is (1) 90

0 (2) 30

0

(3) 600 (4) 120

0

70. In the following figure, if OA = 10 and AC = 16, then OB must be

(1) 5 (2) 6 (3) 3 (4) 4

Directions (71-75) : The pie chart given below shows the distribution of workforce by occupational category for country X in 1981 and 1995. Study the chart and answer the questions.

Total workforce 175 million

71. The increase in the number of Clerical workers in the

workforce of country X from 1981 to 1995 (in millions) is (1) 0.5 (2) 1.25 (3) 0.75 (4) 1.5

72. The percentage decrease in the number of Blue-Collar workers in the workforce of country X from 1981 to 1995 is

(1) 20 (2) 16 2

3

(3) 42 1

2 (4) 35

73. In 1981, the number of Service workers in the workforce, in millions, was

(1) 22.5 (2) 28.0 (3) 15.0 (4) 20.5

74. In 1981, the number of categories which comprised of more than 25 million workers each, is

(1) four (2) five (3) two (4) three

75. The ratio of the number of workers in the Professional category in 1981 to the number of such workers in 1995 is

(1) 9 : 14 (2) 14 : 9 (3) 4 : 9 (4) 5 : 14

English Language Directions (76-80) : In the following questions, some parts of the sentence have errors and some have none, Find out which part of a sentence has an error. The number of that part is your answer. If there is no error, your answer is (4) i.e., No error. 76. The concentration of human and material resources at a

few centres (1) / have resulted in (2) / large scale migration of rural population to urban areas. (3) / No error (4)

TARGET NO-1 CLASSES (A UNIT OF) EXECUTIVE MAKERS PVT.LTD. V-157

FOR DOWNLOADING BOOKS : www.executivemakers.com Page-7

77. Who would’nt be triumphant (1) / in their success (2) / at the examination? (3) / No error (4)

78. I dared (1) / Mohan fights (2) / a duel. (3) / No error (4) 79. All of you except Sheena (1) / have done the homework

(2) / properly. (3) / No error (4) 80. The summit began (1) / on a much voiced (2) / optimism

note. (3) / No error (4) Directions (81-85) : In the following questions, sentences are given with blanks to be filled in with an appropriate word (s). Four alternatives are suggested for each question. Choose the correct alternative out of the four. 81. Many modern methods of agriculture .... thousands of

years ago. (1) sprang (2) started (3) began (4) originated

82. He is utterly ..... at the end of the day. (1) worn away (2) worn on (3) worn through (4) worn out

83. People should refrain ..... evil deeds. (1) from doing (2) doing (3) to do (4) in doing

84. A rare audio tape released by the court at the end of the hearing showed many judges asking .....They should intervene at all.

(1) if only (2) only if (3) weather (4) whether

85. Rita ...... her home work yet. (1) has not done (2) will not do (3) does not do (4) did not do

Directions (86-87) : In the following questions, out of the four alternatives, choose the one which best expresses the meaning of the given word,. 86. Periphery

(1) circle (2) boundary (3) edge (4) terminus

87. Exquisite (1) exact (2) wonderful (3) quizzical (4) exciting

Directions (88-89) : In the following questions, choose the word opposite in meaning to the given word. 88. Nibble

(1) swallow (2) munch (3) devour (4) consume

89. Hindrance (1) assistance (2) authorization (3) approval (4) approbation

Directions (90-92) : In the following questions, four alternatives are given for the Idiom/Phrase printed in bold in the sentence. Choose the alternative which best expresses the meaning of the Idiom/Phrase. 90. Ravi wants to blaze a trail in his activities.

(1) be supported (2) be the centre of attraction (3) initiate work and be a pioneer (4) blow the trumpet

91. She took a wrong decision by not marrying this gentleman. She doesn’t know that beauty is only skin deep.

(1) physical charm is important

(2) internal beauty is not important (3) physical touch is not important (4) physical beauty is not important

92. The officials played ducks and drakes with the public money.

(1) spent (2) preserved (3) collected (4) squandered

Directions (93-95) : In the following questions, out of the four alternatives, choose the one which can be substituted for the given words/sentence. 93. A study of words and the way words develop.

(1) Phraseology (2) Phonography (3) Philology (4) Phonology

94. A list of books. (1) Catalogue (2) Index (3) Anthology (4) Glossary

95. Something unusual or strange, having peculiar notions. (1) Whimsical (2) Aggressive (3) Dramatic (4) Reasonable

Directions (96-100) : In the following questions, a part of the sentence is printed in bold. Below are given alternatives to the bold part at (1), (2) and (3) which may improve the sentence. Choose the correct alternative. In case no improvement is needed your answer is (4). 96. He won’t hear you provided you shout.

(1) in case (2) as long as (3) unless (4) No improvement

97. The ship is at the anchor and the sailors are now at ease. (1) at the anchor (2) at its anchor (3) at anchor (4) No improvement

98. They objected to the plan on the grounds that it was too expensive.

(1) declined (2) disagreed (3) refused (4) No improvement

99. Very little rice was found in the pot after all the guests had eaten.

(1) remained (2) was cooked (3) stored (4) No improvement

100. The policeman captured the first car that approached and ordered the driver to take the injured child to the hospital.

(1) commandeered (2) interrupted (3) caught (4) No improvement

TARGET NO-1 CLASSES (A UNIT OF) EXECUTIVE MAKERS PVT.LTD. V-157

FOR DOWNLOADING BOOKS : www.executivemakers.com Page-8

ANSWER SHEET

1.4 2.1 3.1 4.3 5.4 6.2 7.4

8.1 9.4 10.2 11.3 12.3 13.4 14.2

15.4 16.4 17.4 18.1 19.4 20.4 21.1

22.3 23.4 24.1 25.4 26.4 27.3 28.4

29.1 30.2 31.4 32.3 33.4 34.3 35.2

36.3 37.1 38.1 39.3 40.2 41.2 42.3

43.4 44.3 45.4 46.3 47.4 48.1 49.3

50.3 51.1 52.1 53.3 54.1 55.1 56.4

57.2 58.1 59.1 60.1 61.1 62.4 63.3

64.1 65.3 66.4 67.2 68.1 69.4 70.2

71.4 72.2 73.1 74.4 75.1 76.2 77.2

78.2 79.4 80.3 81.4 82.4 83.1 84.1

85.1 86.3 87.2 88.3 89.1 90.3 91.4

92.4 93.3 94.1 95.1 96.3 97.3 98.4

99.1 100.1

Practice Set – 1

Hints & Solution

1. SKIT ⇒TIKS (Reverse order of letters)

Similarly, PALF ⇒ FLAP.

2. (6)2 = 36, (9)2 = 81.

3. 384 ⇒ 84

3= 28; 2 + 8 = 10; 128 ⇒ 1 + 2 + 8 = 11 =

10 + 1, Similarly, 16

2= 8; 6 + 3 = 9 = 8 + 1.

4. Asthma affects lungs. Similarly, Conjunctivitis affects eyes.

5. Thermometer is used to measure temperature. Similarly,

Glucometer is used to measure blood sugar.

6. Except the number 7586, all other are odd numbers.

7. Except ‘I’ all other are Consonants. Letter ‘I’ is a Vowel.

8. Reed is a type of tall grass with a hollow stem growing near

water. Pit is a large and deep hole. Tap is a small projecting

piece of cloth, metal, paper etc. Hold means to keep or

support. Except in Reed, in all other words, there is only one

Vowel.

9. Z(-4)→V(-4)→R(-4)→N(-4)→J(-4)→F

A(+2)→C(+2)→E(+2)→G(+2)→I(+2)→K.

10. (2)2 = 4, (4)2 = 16, (6)2 = 36, (8)2 = 64, (10)2 = 100.

11. 12 + 11 = 23, 23 + 11 = 34, 34 + 11 = 45, 45 + 11 =

56, 56 + 11 = 67,67 + 11 = 78, 78 + 11 = 89.

12. A(+1)→B(+5)→G(+1)→H

C(+1)→D(+5)→I(+1)→J

E(+1)→F(+5)→K(+1)L

13. D(+4)→H(+5)→M(+6)→S

E(+4)→I(+5)→N(+6)→T

F(+4)→J(+5)→O(+6)→U

14. 2 + 0 + 1 + 5 = 8; 8 × 3 = 24, 6 + 4 + 1 + 3 = 14; 14 ×

3 = 42, 1 + 1 + 2 + 8 = 12; 12 × 3 = 36.

15.

Required distance = AE = AD +DE = (10 + 2) km=12 km.

16.

17. Option (a) 24 = 4 > 5 × 17 + 13 ⇒ 24 ÷ 4 + 5 = 17 −

13 ⇒ 6 + 5 ≠ 4 Option b 24 > 4 + 5 × 17 + 13 ⇒ 24 +

4 − 5 = 17 − 13 ⇒ 23 ≠ 4, option c 24 = 4 > 5 × 17 >

13 ⇒ 24 ÷ 4 + 5 = 17 + 13 ⇒ 6 + 5 ≠ 30.

18. First row 5 × 7 + 4 = 35 + 4 = 39 Second row 6 × 9 + 5 =

54 + 5 = 59 Third row 7 × 11+? = 83 ⇒? = 83 − 77 = 6.

19. First column 8 ÷ 2 = 4 𝑎𝑛𝑑 8 − 4 = 4 second column

16 ÷ 8 = 2 and 5 − 3 = 2 Third column 10 ÷ 5 = 2 and

11 − 9 = 2.

20. First row 15 − 8 = 7 and 7 × 2 = 14, Second row 23 −

13 = 9 and 9 × 2 = 18, Third row 41 − 25 = 16 and 16 ×

2 = 32.

21. Do yourself.

22. Do yourself.

23. Some hockey players may be poet and vice-versa. Rubber ball

is different from the two.

24.

Squares are: ABCD, DEFG, HIJK, LMNO, HPDT, TDSK, PIWD,

DWJS, LQDU, UDRO, QMVD, DVNR.

25. Only conclusion II follows. It is mentioned clearly that

parental role in the education is very significant.

51. Let marked price of the book be Rs. x and C.P. Rs. y. 90x

100=

y×108

100⇒

y

x=

90

108= 5: 6.

52. Amount received by B = 5

2× 100 = Rs. 250.

53. (2.1)2 × 0.0441 = 2.1 × 2.1 × 0.21 = 0.9261.

54. Required number = HCF of 140 − 4 = 136; 176 − 6 =

170; 𝑎𝑛𝑑 264 − 9 = 255 = 17.

TARGET NO-1 CLASSES (A UNIT OF) EXECUTIVE MAKERS PVT.LTD. V-157

FOR DOWNLOADING BOOKS : www.executivemakers.com Page-9

55. 10 men = 18 boys, 15 men = 27 boys

15 men + 33 boys = 27+33= 60 boys.

M1D1

W1=

M2D2

W2⇒

18×15

1=

60×D2

2,

D2 = 18×15×2

60= 9 days.

56. 13723

× 14583

= 1372 × 14583

=

2 × 2 × 7 × 7 × 7 × 2 × 9 × 9 × 93

= 2 × 7 × 9 = 126.

57. If the altitude be x cm. then 1

2× 2x + 2 x = 12 ⇒

x + 1 x = 12 ⇒ X2 + x − 12 = 0 ⇒ X2 + 4x − 3x − 12 =

0 ⇒ x x + 4 − 3 x + 4 = 0 ⇒ x = 3 because x ≠ −4.

58. Amount to each nephew = Rs. x. amount to each daughter =

Rs. 3x. Amount to each son = 7x , 7 × 7𝑥 + 3 × 3𝑥 + 5𝑥 =

12600 ⇒ 49𝑥 + 9𝑥 + 5𝑥 = 12600 ⇒ 63𝑥 = 12600 ⇒ 𝑥 =

12600 ÷ 63 ⇒ 𝑥 = 200.

Amount to each daughter = 200 × 3 =Rs. 600.

59. C.P. of article = 100

105× 21000 = Rs. 20000 , S. P. for 15% profit = 20000 ×

115

100= Rs. 23000.

60. Required salary = 15000(1 +10

100)2 = 15000 ×

11

10×

11

10=

𝑅𝑠. 18150.

61. Volume of sphere = curved surface of sphere ⇒ 4

3𝜋𝑟3 =

4𝜋𝑟2 ⇒ 𝑟 = 3 𝑢𝑛𝑖𝑡𝑠.

62. x2 ≥ 0, Minimum value = 0+1

1− 3 = −2.

63. 3x−y

x+5y=

5

7⇒ 21x − 7y = 5x + 25y ⇒ 21x − 5x = 25y +

7y ⇒ 16x = 32y ⇒x

y=

32

16=

2

1⇒

x+y

x−y=

2+1

2−1= 3: 1.

[ By componendo and dividendo].

64.

B=60°; C=40°, A=180°-100°=80° ,BAD=DAC=40°

From ∆ABE, BAE = 180°-60°-90°=30°, EAD= 40°-30°=10°.

65. πR2h = 2πRh ⇒ R = 2 units.

66.

tan B=AC

AB=

b

c; tan C=

AB

AC=

c

b, tan B + tan C =

b

c+

c

b=

b2+c2bc=a2bc.

67. tan 7°. tan 23°.tan 60°tan 67°.tan 83°

=tan 7°.tan 83°. tan 23°. tan 67°. tan 60°

=tan 7°. tan (90°-7°). tan 23°. tan (90°-23°). tan 60°

=tan 7°.cot 7°.tan 23°. cot 23°.tan 60°=1. 3 = 3.

[tan (90°-𝚹)=cot 𝚹. tan 𝚹cot 𝚹=1].

68. (sec 𝚹 - cos 𝚹)(cosec 𝚹 –sin 𝚹)(tan 𝚹+cot 𝚹)=(1

cos θ−

cosθ)(1

sin θ−

sin θ) sin θ

cos θ+

cos θ

sin θ =

1−cos 2θ

cos θ

1−sin 2θ

sin θ

sin 2θ+cos 2θ

sin θ.cos θ =

sin 2θ

cos θ.

cos 2θ

sin θ

1

sin θ.cos θ = 1.

69.

AG=BG=GC AGB=AGC=BGC=120°.

70.

AB=BC=8 , OA=10

OB= OA2 − AB2 = 102 − 82 = 36 = 6.

71. Number of clerical workers in 1981 = 150×20

100= 30 million,

Number of clerical workers in 1995 = 175×18

100= 31.5 million

Difference =31.5 – 30 = 1.5 million.

72. Number of blue-collar workers in 1981 = 150×28

100=

42 million,

Number of blue − collar workers in 1991 =175×20

100=

35 million

Percentage decrease = 42−35

42× 100 =

50

3= 16

2

3.

73. Service workers in 1981= 150×15

100= 22.5 million.

74. 150×x

100= 25 ⇒ x =

25×100

150= 16

2

3%. Required answer = blue-

collar, professional and clerical workers.

75. Required Ratio = 150×18

100:

175×24

100= 6 × 3: 7 × 4 = 9 × 14.

76. Here, subject (The concentration) is singular. Hence, singular

verb i.e., has resulted in ….. should be used.

77. Here, at/over ,his/their success ….. should be used.

78. Here, Mohan fight (infinitive without to )… should be used.

80. Here, optimistic (Adjective) note …..Should be used.

82. Phrase wear yourself/somebody out means: to make

yourself/somebody feel very tired.

86. The word Periphery (Noun) means: the outer edge of a

particular area.

Look at the sentence: Industries have been established on

the periphery of the town.

87. The word Exquisite (Adjective) means: extremely beautiful or

carefully made: acute, delicate and sensitive; wonderful.

TARGET NO-1 CLASSES (A UNIT OF) EXECUTIVE MAKERS PVT.LTD. V-157

FOR DOWNLOADING BOOKS : www.executivemakers.com Page-10

88. The word Nibble (Verb) means: to make small bites of

something. The word Devour (Verb) means: to eat all of

something quickly, engulf: to destroy.

89. The word Hindrance (Noun) means: the act of making it more

difficult for somebody to something. Its antonym should be

assistance which means: help or support.

90. Idiom blaze a trail means: to be the first to do or discover

something that others follow.

92. Idiom play ducks and drakes means: squander; to waste

money, time etc. in a stupid or carless way.

PRACTICE SET-2 General Intelligence & Reasoning

Directions (1-5) : In each of the following questions, select the related letters/words/number from the given alternatives. 1. 8 : 12 : : 6 : ?

(1) 8 (2) 11 (3) 5 (4) 7

2. Eagle : Swoops : : Duck : ? (1) Waddles (2) Floats (3) Swims (4) Flits

3. Fire : Smoke : : ? (1) Children : School (2) Cloud : Rain (3) Moon : Sky (4) Shoe : Polish

4. APPLE : 50 : : ORANGE : ? (1) 60 (2) 69 (3) 61 (4) 63

5. TSH : IRQ : : QPK : ? (1) LNO (2) LON (3) PWK (4) PON

Directions (6-8) : In each of the following questions, find the odd number/letters/word/ number pair from the given alternatives. 6. (1) SP (2) NL (3) ZW (4) TQ 7. (1) Major (2) Colonel (3) Brigadier (4) Admiral 8. (1) (132, 5) (2) (125, 8) (3) (124, 7) (4) (112, 4) Directions (9-11) : In each of the following questions, a series is given, with one term missing. Choose the correct alternative from the given ones that will complete the series. 9. YX, UTS, ONML, ?

(1) FEDCB (2) CFEDC (3) IHGFE (4) HGFED

10. AZ, CX, EV, ? (1) HT (2) HU (3) GS (4) GT

11. Which one set of letters when sequentially placed at the gaps in the given letter series shall complete it?

....C....bd....cbcda....a....db...a (1) daabbc (2) bdbcba (3) adabcd (4) cdbbca

Directionss (12-13) : In each of the following questions, identify the wrong number in the series.

12. 9, 19, 40, 83, 170, 340 (1) 83 (2) 40 (3) 340 (4) 170

13. 5, 13, 29, 61, 120, 253 (1) 120 (2) 253 (3) 61 (4) 29

14. Some relationships have been expressed through symbols which are explained below:

= greater than

= not equal to

= not less than

= equal to

= not greater than

= less than

a b c implies

(1) a b c (2) a b + c

(3) a b + c (4) a b c 15. If PEAR is written as GFDN, how is REAP written in this

code? (1) FDNG (2) NFDG (3) DNGF (4) NDFG

16. If L denotes , M denotes , P denotes +, Q denotes – then 16 P 24 M 8 Q 6 M 2 L 3 = ?

(1) 10 (2) 9 (3) 12 (4) 11

17. If FLATTER is coded as 7238859 and MOTHER is coded as 468159, then how is MAMMOTH coded?

(1) 4344681 (2) 4344651 (3) 4146481 (4) 4346481

Directions (18-19) : In each of the following questions, select the missing number from the given responses. 18.

(1) 20 (2) 25 (3) 10 (4) 15

19.

10 11 15

12 12 8

4 12 10

10 5 13

18 20 ?

(1) 21 (2) 20 (3) 23 (4) 22

20. Satish starts from A and walks 2 km East upto B and turns Southwards and walks 1 km upto C. At C he turns to East and walks 2 km upto D. He then turns Northwards and walks 4 km to E. How far is he from his starting point?

TARGET NO-1 CLASSES (A UNIT OF) EXECUTIVE MAKERS PVT.LTD. V-157

FOR DOWNLOADING BOOKS : www.executivemakers.com Page-11

(1) 5 km (2) 6 km (3) 3 km (4) 4 km

Directions (21) : In each of the following questions, one/two statements are given, followed by two conclusions I and II. You have to consider the statements to be true, even if they seem to be at variance from commonly known facts. You have to decide which of the given conclusions, if any follow from the given statement. 21. Statements : Temple is a place of worship.

Church is also a place of worship Conclusions : I. Hindus and Christians use the same place for worship. II. All churches are temples.

(1) Neither conclusion I nor II follows (2) Both conclusion I and II follows (3) Only conclusion I follows (4) Only conclusion II follows

22. In the following letter series, how many times do PQR occur in such away that Q is in the middle of P and R. Q M P N P Q R R O P Q N O P P Q R P M Q R O P Q R P P R R P Q R P

(1) 5 (2) 6 (3) 4 (4) 3

23. A sheet of paper has been folded as shown by the question figures. You have to figure out from amongst the four answer figures how it will appear when opened? Question Figures :

Answer figures :

(1) (2)

(3) (4) 24. Which of the answer figure is exactly the mirror-image of

the question figure if a mirror is placed on the line MN? Questions Figure

Answer Figures :

(1) (2)

(3) (4) 25. From the answer figures, select the one in which the

question figure is hidden/embedded. Question Figure :

Answer Figure :

(1) (2)

(3) (4)

General Awareness

26. The National Commission for Minorities was constituted in the year

(1) 1990 (2) 1992 (3) 1980 (4) 1989

27. The first Indian who was chosen as the Secretary General of Common-wealth is

(1) Rakesh Verma (2) Gopalaswami (3) Krishna Murthy (4) Kamalesh Sharma

28. Socialism succeeds in achieving (1) higher standard of living of the people (2) equal distribution of income in the society (3) higher individual welfare in the society (4) maximum social welfare in the society

29. Economic planning is an essential feature of (1) socialist economy (2) capitalist economy (3) mixed economy (4) dual economy

30. Which of the following criteria is not used for the classification of human races?

(1) Nose (2) Hair (3) Eyes (4) Ear

31. Railway coaches are manufactured at (1) Jamshedpur (2) Chittaranjan (3) Perambur (4) Varanasi

32. Fertilization occurs normally in the (1) cervix (2) vagina (3) fallopian tube (4) uterus

33. The organisms at the base of the grazing food-chain are (1) carnivores (2) decomposers (3) producers (4) herbivores

34. Who among the following was credited with the destruction of ‘Chihalgani’, a group of powerful nobles?

(1) Balban (2) Qutb-ud-din Aibak (3) Iltutmish (4) Razia Sultan

35. The Maratha Saint who was a contemporary of Shivaji (1) Saint Eknath (2) Saint Tukaram (3) Saint Dhyaneshwar (4) Namdev

36. The study of lake is called (1) Topology (2) Hydrology (3) Limnology (4) Potomolgy

37. ‘Lumen’ is the unit of (1) Illuminance (2) Brightness (3) Luminous intensity (4) computer ports

TARGET NO-1 CLASSES (A UNIT OF) EXECUTIVE MAKERS PVT.LTD. V-157

FOR DOWNLOADING BOOKS : www.executivemakers.com Page-12

38. Which of the following items is not used in Local Area Networks (LANs)?

(1) Interface Card (2) Cable (3) Computer (4) Modem

39. Vitamin A is rich in (1) carrot (2) lime (3) beans (4) rice

40. Which of the following determines the chemical properties of an element?

(1) Number of electrons (2) Number of neutrons (3) Number of protons (4) All of the above

41. The Central Drug Research Institute of India is located at (1) Chennai (2) Lucknow (3) Delhi (4) Bangalore

42. Which one of the following forces is a ‘dissipative force’? (1) Electrostatic force (2) Magnetic force (3) Gravitational force (4) Frictional force

43. How many spokes are there in the Dharmachakra of the National Flag?

(1) 14 (2) 18 (3) 22 (4) 24

44. The latest official language of the U.N. is (1) Russian (2) Arabic (3) Chinese (4) Spanish

45. The book ‘Kurukshetra to Kargil’ is written by (1) Suryanath Singh (2) Kunal Bhardwaj (3) Karan Singh (4) Kuldip Singh

46. The sweet taste of fruits is due to (1) Lactose (2) Fructose (3) Maltose (4) Ribose

47. ‘Analects’ is the sacred book of (1) Confucianism (2) Judaism (3) Shintoism (4) Taoism

48. What would be the impact of global warming on mangrove forests?

(1) They will grow more luxurious (2) Large areas of mangroves will be submerged (3) Their role as carbon sinks will become more important (4) Both (1) and (3) above

49. the seat of Madhya Pradesh High Court is located at (1) Gwalior (2) Indore (3) Bhopal (4) Jabalpur

50. The brightest planet is (1) Venus (2) Mercury (3) Jupiter (4) Mars

Quantitative Aptitude

51. P and Q are two points observed from the top of a

building 10 3 m high. If the angles of depression of the points are complementary and PQ = 20 m, then the distance of P from the building is

(1) 25 m (2) 45 m (3) 30 m (4) 40 m

52. the least value of 2 sin2 + 3 cos

2 is

(1) 3 (2) 5

(3) 1 (4) 2 53. A, O, B are three points on a line segment and C is a point

not lying on AOB. If AOC = 400 and OX, OY are the

internal and external bisectors of AOC respectively,

then BOY is (1) 70

0 (2) 80

0

(3) 720 (4) 68

0

54. If 4x = sec and 4

𝑥 = tan , then 8 𝑥2 −

1

𝑥2 is

(1) 1

16 (2)

1

8

(3) 1

2 (4)

1

4

55. If x sin3 + y cos

3 = sin cos and x sin = y cos , sin

0. cos 0, then x2 + y

2 is

(1) 1

2 (2)

1

2

(3) 1 (4) 2

56. The side BC of ABC is produced to D. If ACD = 1080

and B = 1

2 A, then A is

(1) 360 (2) 72

0

(3) 1080 (4) 59

0

57. Two tangents are drawn from a point P to a circle at A

and B .O is the centre of the circle. If AOP = 600, then

APB is (1) 120

0 (2) 90

0

(3) 600 (4) 30

0

58. The angle formed by the hour-hand and the minute-hand of a clock at 2 : 15 p.m. is

(1) 27 1°

2 (2) 45

0

(3) 22 1°

2 (4) 30

0

59. If x = (0.08)2, y =

1

(0.08)2 and z = (1-0.08)2 – 1, then out of

the following, the true relation is (1) y < x and x = z (2) x < y and x = z (3) y < z < x (4) z < x < y

60. If 𝑥4 +1

𝑥4 = 23 , then the value of 𝑥 −1

𝑥

2

will be

(1) 7 (2) -7 (3) -3 (4) 3

61. If 𝑥 +1

𝑥= 3 , the value of 𝑥5 +

1

𝑥5 is

(1) 123 (2) 126 (3) 113 (4) 129

62. In ABC, AD is the median and AD = 1

2 BC. If BAD = 30

0,

then measure of ACB is (1) 90

0 (2) 45

0

(3) 300 (4) 60

0

63. 3 −3+ 5

4−

1

3+ 5 is equal to

(1) 0 (2) 3

2

(3) 5

2 (4) 5

64. If the average of 39, 48, 51, 63, 75, 83, x and 69 is 60, then the value of x is

(1) 52 (2) 53 (3) 50 (4) 51

65. The cost of a piece of diamond varies with the square of its weight. A diamond of Rs. 5,184 value is cut into 3 pieces whose weights are in the ratio 1 : 2 : 3. Find the loss involved in the cutting.

(1) Rs. 3,068 (2) Rs. 3,088

TARGET NO-1 CLASSES (A UNIT OF) EXECUTIVE MAKERS PVT.LTD. V-157

FOR DOWNLOADING BOOKS : www.executivemakers.com Page-13

(3) Rs. 3,175 (4) Rs. 3,168 66. A discount of 30% on the marked price of a toy reduces

its selling price by Rs. 30. What is the new selling price (in Rs)?

(1) 70 (2) 21 (3) 130 (4) 100

67. Pipe A alone can fill a tank in 8 hours. Pipe B alone can fill it in 6 hours. If both the pipes are opened and after 2 hours pipe A is closed, then the other pipe will fill the tank in

(1) 6 hours (2) 3 1

2 hours

(3) 4 hours (4) 2 1

2 hours

68. If Rs. 5,000 becomes Rs. 5,700 in a year’s time, what will Rs. 7,000 become at the end of 5 years at the same rate of simple interest?

(1) Rs. 10,500 (2) Rs. 11,900 (3) Rs. 12,700 (4) Rs. 7,700

69. The number nearest to 75070 which is divisible by 65, is (1) 75070 (2) 75075 (3) 75010 (4) 75065

70. A farmer divides his herd of n cows among his four sons, so that the first son gets one-half the herd, the second

one-fourth, the third son 1

5 and the fourth son 7 cows.

Then the value of n is (1) 240 (2) 100 (3) 180 (4) 140

Directions (71-75) : The graph shows Income and Expenditure of a company. Study the graph and answer the questions.

71. The expenditure from 2002 to 2003 increased by

(1) 33 1

3% (2) 40%

(3) 10% (4) 20% 72. The income in 2002 was equal to the expenditure in the

year (1) 2003 (2) 2004 (3) 2000 (4) 2001

73. The profit was maximum in the year (1) 2003 (2) 2004 (3) 2001 (4) 2002

74. The difference in profit between 2001 and 2002 is (1) Rs. 25 lakh (2) No difference (3) Rs. 10 lakh (4) Rs. 20 lakh

75. The number of years in which the income exceed the average income is

(1) three (2) four (3) one (4) two

English Language

Directions (76-80) : In the following questions, some parts of the sentences have errors and some have none. Find out which part of a sentence has an error. If a sentence is free from error, then your answer is (4) i.e., No error. 76. You do not (1) / look as (2) / your brother. (3) / No error

(4) 77. My elder brother (1) / is six (2) / foot high. (3) / No error

(4) 78. Without no proof of your guilt (1) / the only course open

to me (2) / is to dismiss the case. (3) / No error (4) 79. As we see it, (1) / she appears to be unreasonable (2) /

anxious about pleasing her husband. (3) / No error (4) 80. The scissor is (1) / lying on (2) / the table. (3) / No error

(4) Directions (81-82) : In the following questions, out of the four alternatives, choose the one which best expresses the meaning of the given word as your answer. 81. Nexus

(1) connection (2) distance (3) deficit (4) difference

82. Hyperbole (1) expansion (2) imitation (3) decoration (4) exaggeration

Directions (83-84) : In the following questions, choose the word opposite in meaning to the given word as your answer. 83. Amalgamate

(1) separate (2) combine (3) assimilate (4) integrate

84. Influx (1) reflex (2) deflection (3) effluent (4) exodus

Directions (85-87) : In the following questions, four alternatives are given for the Idiom/Phrase printed in bold in the sentence. Choose the alternative which best expresses the meaning of the Idiom/Phrase as your answer. 85. Ram is very calculative and always has an axe to grind.

(1) has no result (2) works for both sides (3) has a private agenda (4) fails to arouse interest

86. On the issue of marriage, Sarita put her foot down. (1) stood up (2) was firm (3) got down (4) walked fast

87. There is no gainsaying the fact that the country is in difficulties.

(1) ignoring (2) hiding (3) forgetting (4) denying

Directions (88-90) : In the following questions, out of the four alternatives, choose the one which can be substituted for the given words/sentence. 88. Someone able to use both hands with equal skill.

(1) Ambivalent (2) Amphibious (3) Ambiguous (4) Ambidextrous

89. Cure for all diseases. (1) Curable (2) Panacea (3) Incurable (4) Curative

TARGET NO-1 CLASSES (A UNIT OF) EXECUTIVE MAKERS PVT.LTD. V-157

FOR DOWNLOADING BOOKS : www.executivemakers.com Page-14

90. Speaking without preparation. (1) Deliberate (2) Fluent (3) Loquacious (4) Extempore

Directions (91-95) : In the following questions. a part of the sentence is printed in bold. Below are given alternatives to the bold part at (1), (2) and (3) which may improve the sentence. Choose the correct alternative. In case no improvement is needed, your answer is (4). 91. Sordid and sensational books tend to vitiate the public

taste. (1) divide (2) distract (3) distort (4) No improvement

92. By studying AIDS has engaged many researchers in the last decade.

(1) Important study (2) Now that the study (3) The study of (4) No improvement

93. His Master’s thesis was highly estimated and is now being prepared for publication.

(1) was highly discussed. (2) was highly commended. (3) is highly appraised. (4) No improvement

94. No sooner had she realized her blunder than she began to take corrective measures.

(1) then she began to take (2) then she began taking (3) when she began to take (4) No improvement

95. A good scholar must be precise and possess originality. (1) must be precise and original. (2) must be possess precision and original. (3) must be precision and possess originality. (4) No improvement

Directions (96-100) : In the following questions, you have two brief passages with 5 questions following each passage. Read the passage carefully and choose the best answer to each question out of the four alternatives.

Passage-1 Two years later, in November 1895, he signed his final will. He left the bulk of his fortune, amounting to about £ 1,75,000 to a trust fund administered by Swedish and Norwegian trustees. The annual interest shall be awarded as prizes to those persons who during the previous year have rendered the greatest services to mankind. The interest shall be divided into five equal parts – now amounting to about £ 8,000 each – one of which shall be awarded to the person who has made the most important discovery or invention in the realm of physics, one to the person who has made the most important chemical discovery or improvement, one to the person who has made the most important physiological or medical discovery, one to the person who has produced the most outstanding work of literature, idealistic in character and one to the person who has done the best work for the brotherhood of nations, the abolition or reduction of standing armies, as well as for the formation or popularization of peace congress.

96. The said prize is awarded

(1) once in 5 years (2) every year (3) once in 4 years (4) once in 2 years

97. Which is the prize that is referred to in the passage? (1) Noble Prize (2) Magsaysay Award (3) Pulitzer Prize (4) Booker Prize

98. The number of prizes in the field of science are (1) four (2) one (3) three (4) five

99. Total annual price money amounts to (1) £ 8,000 (2) £ 1,750,000 (3) £ 350,000 (4) £ 40,000

100. Prize is awarded for outstanding work in (1) Chemistry (2) Literature (3) Physics (4) All of these

ANSWER SHEET

1.1 2.1 3.2 4.1 5.2 6.2 7.4

8.4 9.2 10.4 11.3 12.3 13.1 14.1

15.2 16.1 17.1 18.4 19.3 20.1 21.1

22.3 23.2 24.4 25.4 26.2 27.4 28.2

29.1 30.4 31.3 32.3 33.3 34.1 35.2

36.3 37.3 38.3 39.1 40.1 41.2 42.4

43.4 44.2 45.4 46.2 47.1 48.4 49.4

50.1 51.3 52.4 53.1 54.3 55.3 56.2

57.3 58.3 59.4 60.4 61.1 62.4 63.2

64.1 65.4 66.1 67.4 68.2 69.2 70.4

71.1 72.1 73.2 74.3 75.1 76.2 77.3

78.1 79.2 80.1 81.1 82.4 83.1 84.4

85.3 86.2 87.4 88.4 89.2 90.4 91.2

92.3 93.2 94.4 95.1 96.2 97.1 98.3

99.1 100.4

Practice Set-2

Hints & Solution 1. The relationship is 𝑥: 2𝑥 − 4

8 × 2 − 4 = 16 − 4 = 12, Similarly, 6 × 2 − 4 = 12 − 4 = 8.

2. Eagle swoops down on anything. The movement of eagle is

like swooping. Similarly, the movement of duck is called

waddle.

3. Fire causes smoke. Smoke comes out when something is

burnt in fire. Similarly, cloud causes rain.

4. APPLE = 1+16+16+12+5 = 50

5. T (-2) R, S (-2) Q, H (+1) I

6. S(-3)P, N(-2)L, Z(-3)W, T(-3)Q.

7. Major, Colonel and Brigadier are different ranks in the Indian

Army. Admiral is the topmost rank in the Indian Navy.

8. 112 is completely divisible by 4. 112

4= 28,

132

5= 26.4,

125

8=

15.625 and 124

7= 17.71.

9. Y (-1) X,X (-3) U, U(-1)T, T(-1)S, S(-4)O,O(-1)N,N(-1)M……..

10. A(+2)→C(+2)→E(+2)→G.

Z(-2)→X(-2)→V(-2)→T

11. a c d b/ d a c b / c d a b / a c d b / d a

TARGET NO-1 CLASSES (A UNIT OF) EXECUTIVE MAKERS PVT.LTD. V-157

FOR DOWNLOADING BOOKS : www.executivemakers.com Page-15

12. 9 × 2 + 1 = 18 + 1 = 19,

19 × 2 + 2 = 38 + 2 = 40,

40 × 2 + 3 = 80 + 3 = 83,

83 × 2 + 4 = 166 + 4 = 170,

170 × 2 + 5 = 340 + 5 = 𝟑𝟒𝟓.

13. 5 + 8 = 13, 13 + 16 = 29, 29 + 32 = 61, 61 + 64 =

𝟏𝟐𝟓, 125 + 128 = 253.

14. Do yourself.

15. P→G, E→F, A→D, R→N Therefore, R→N, E→F, A→D, P→G.

16. ----- 16P24M8Q6M2L3=? ⇒ ?=16+24÷8 -6÷2×3⇒ ?=16+3-

3×3⇒ ?=16+3-9=10.

17. F→7,L→2, A→3, T→8, T→8, E→5, R→9

M→4, O→6, T→8, H→1, E→5, R→9,

Therefore, M→4, A→3, M→4, M→4, O→6, T→8, H→1

18. The product of two numbers in a sector is equal to the central

number in the previous sector.3 × 5 = 15, 8 × 3 = 24, 7 ×

2 = 14, 5 × 3 = 15, 8 × 4 = 32, 9 × 1 = 9, 9 × 2 = 18, 7 ×

4 = 28.

19. First column 10 + 12 + 4 + 10 = 36,36

2= 18 =

18 lowermost number

𝐒𝐞𝐜𝐨𝐧𝐝 𝐜𝐨𝐥𝐮𝐦𝐧 11 + 12 + 12 + 5 = 40,40

2= 20

𝐓𝐡𝐢𝐫𝐝 𝐜𝐨𝐥𝐮𝐦𝐧 15 + 8 + 10 + 13 = 46,46

2= 23.

20.

Required distance (AE) = (AF)2 + (EF)2 = (4)2 + (3)2 =

16 + 9 = 25 = 5 km.

21. Temple and Church are places of worship. It does not imply

that Hindus and Christians use the same place for worship.

Church is different temple. Therefore, neither Conclusion I

nor II follows.

22. QMPN PQR ROPQNOP PQR MQRO PQR PPRR PQR P

25. Do yourself.

26. The Union Government set up the National Commission for

Minorities (NCM) under the National commission for

Minorities Act, 1992.

27. Mr. Kamalesh Sharma, an Indian diplomat, became

Commonweatlth Secretary-General on 1 April, 2008. He was

appointed to the post by Commonwealth Heads of

Government at their meeting in Kampala, Uganda, in

November 2007.

28. Socialism is an economic system characterized by social

ownership and/or control of the means of production and

cooperative management of the economy.

29. Economic planning is an essential feature of socialism. The

most prominent example of a planned economy was the

economic system of the Soviet Union.

30. Ear is not used as criteria for racial classification.

31. Passenger coaches are manufactured at three principal

places: Integral Coach Factory (ICF) at Perambur, Railway

Coach Factory (RCF) at Kapurthala, and Bharat Earth Movers

Ltd, (BEML) at Bangalore.

32. This process usually occurs in the upper third of the fallopian

tube of the woman.

33. The food webs we see are grazing food chains since at their

base are producers which the herbivores then graze on.

34. Chihalgani was the group of most important and powerful

forty nobles or highly placed officers in the court of Iltutmish.

Balban destroyed their clout.

35. Shivaji, the great Maratha King and founder of a nationalist

tradition was contemporary of Tukaram.

36. The study of lakes and other freshwater basins is known as

limnology.

37. The lumen is the SI derived unit of luminous flux, a measure

of the total “amount” of visible light emitted by a source.

38. Interface Card, Ethernet Cable and Routers are used in setting

up a LAN. The Router can be plugged directly into the modem

via an Ethernet cable and all other computers are eventually

connected in some fashion to the route. In terms of LAN, a

computer cannot be said to be an item which constitutes this

network.

39. Carrot is rich in Vitamin-A and it improves eyesight.

40. Valence electrons are important in determining how an

element reacts chemically with other elements. Since the

valence electrons are the electrons in the highest energy

level, they are the most exposed of all the electrons, so they

are the electrons that get most involved in chemical

reactions.

41. Inaugurated in 1951 by the Prime Minister of India. Jawahar

Lal Nehru, CDRI is located at Lucknow.

42. A dissipative force counteracts motion. Its direction is

opposite to the direction of the velocity vector. Dynamic

friction is a dissipative (non-conservative) force: it dissipates

energy (mainly through heat and sound), and energy lost by

moving in one direction.

43. The spokes in the Ashok Chakra represent 24 States as 24

were the number of States at that time.

44. In 1980, the General Assembly made Arabic an official and

working language of all its committees and subcommittees.

45. The author is Brigadier (Retired) Kuldip Singh.

46. Fructose, or fruit sugar, is a simple monosaccharide found in

many plants. It is one of the three dietary monosaccharide,

along with glucose and galactose, which is absorbed directly

into the blood-stream during digestion.

47. The Analects, also known as the Analects of Confucius; is the

collection of sayings and ideas attributed to the Chinese

philosopher Confucius and his contemporaries, traditionally

believed to have been written by Confucius followers.

TARGET NO-1 CLASSES (A UNIT OF) EXECUTIVE MAKERS PVT.LTD. V-157

FOR DOWNLOADING BOOKS : www.executivemakers.com Page-16

48. Global warming may promote expansion of mangrove forests

to higher latitudes, Elevated CO2 concentration may increase

mangrove growth by stimulating photosynthesis or improving

water use efficiency. Mangrove wetlands are possible

sinks/sources for carbon dioxide and other related

greenhouse gases.

49. The Court was established in Nagpur, but after the

reorganization of States on 1 November, 1956, it was moved

to Jabalpur.

50. Venus is very bright. That’s partly because sunlight is easily

reflected by acidic clouds that blanket the planet’s

atmosphere.

51.

AB= Building = 10 3 metre , PQ = 20 metre , BQ =

x metre let If APB = θ, then, AQB = 90° −

θ, From ∆ABP, tan θ =AB

BP=

103X+20……..1 From ∆ABQ,

tan90°−θ=ABBQ⇒cotθ=103x……..2By multiplying both

equations,tanθ.cotθ=103x+20×103x⇒x2+20x=10×10×3

⇒x2+20x−300=0⇒xx+30−10x+30=0⇒x−10x+30=0⇒x

=10, x≠−30 ,BP=10+20=30 metre.

52. 2sin2θ + 3cos2θ = 2sin2θ + 2cos2θ + cos2θ =

2 sin2θ + cos2θ + cos2θ = 2 + cos2θ , Least value = 2 +

0 = 2 cos2θ ≥ 0 .

53.

OY is the bisector of AOC, AOC = 2 COX ,

OX is the bisector of BOC , BOC = 2 COY,

AOC + BOC = 2 COY +2 COX = 180°

⇒2(COX+YOC)=180°

⇒XOY = 90° AOX +XOY + BOY = 180°,

BOY = 180°-90°-20°=70°.

54. 4𝑥 = sec θ ⇒ x =sec θ

4 Again,

4

x= tan θ ⇒

1

x=

tan θ

4, 8 x2 −

1x2=8sec2θ16−tan2θ16=816sec2θ−tan2θ=12.

55. xsin3θ + ycos3θ = sin θ. cos θ ⇒ (x sin θ). sin2θ +

(x sin θ). cos2θ = sin θ. cos θ ⇒ xsin θ. (sin2θ + cos2θ) =

sin θ. cos θ ⇒ x = cos θ , xsin θ = y cos θ ⇒ cos θ. sin θ =

ycos θ ⇒ y = sin θ, x2 + y2 = sin2θ + cos2θ = 1.

56.

ACD = ABC + BAC⇒108°=A

2+ A ⇒

3A

2= 108° ⇒

A =108×2

3=72°.

57.

In right ∆s OAP and OPB

AP=PB, OA = OB, OP = OP, ∆OAP ≅ ∆OPB,

AOP = POB and APO = OPB

From ∆AOP,APO = 180° − 90° − 60° = 30°.

APB = 2 × 30° = 60°.

58. Angle traced by hours hand in an hours = 30°

Angle traced in 21

4i. e.

9

4hours =

9

4× 30° =

135°

2

Angle traced by minute hand in 60 minutes = 360°

Angle traced in 15 minutes = 360°

60× 15 = 90°

Required angle = 90°-135°

2=

45

2= 22

1

2°.

59. 𝑥 = 0.08 2, 𝑦 =1

0.08 2 =10000

64= 156.25 ,

𝑧 = (1 − 0.08)2 − 1 = 1 + (0.08)2 − 2 × 0.08 − 1

= (0.08)2 − 2 × 0.08 clearly, z < 𝑥 < 𝑦.

60. 𝑥4 +1

𝑥4 = 23 ⇒ (𝑥2 +1

𝑥2)2 − 2 = 23 ⇒ (𝑥2 +1

𝑥2)2 = 23 +

2 = 25 , 𝑥2 +1

𝑥2 = 5, . (𝑥 −1

𝑥)2 = 𝑥2 +

1

𝑥2 − 2 = 5 − 2 = 3.

61. 𝑥 +1

𝑥= 3 on Squaring, x +

1

x

2

= 9 ⇒ 𝑥2 +1

𝑥2 = 9 − 2 =

7 Again, (x +1

x)3 = 27 ⇒ x3 +

1

x3 + 3 x +1

x = 27 ⇒ x3 +

1

x3 = 27 − 3 × 3 = 18

x2 +1

x2 x3 +1

x3 ⇒ x5 +1

x5 + x +1

x = 126 ⇒ x5 +

1

x5 =

126 − 3 = 123.

62.

BD= DC =AD , BAD = 30° From ∆ABD, BAD = 30°

ABD= BAD= 30°, ADB = 180°-2×30°=120°, ADC = 180°-

120°=60° ,AD = DC ⇒ DAC = ACD = 60°.

63. 1

3+ 5=

3− 5

3+ 5 3− 5 =

3− 5

9−5=

3− 5

4 ,

3 −3+ 5

4−

3− 5

4=

12−3− 5−3+ 5

4=

6

4=

3

2.

TARGET NO-1 CLASSES (A UNIT OF) EXECUTIVE MAKERS PVT.LTD. V-157

FOR DOWNLOADING BOOKS : www.executivemakers.com Page-17

64. 39 + 48 + 51 + 63 + 75 + 83 + 𝑥 + 69 = 60 × 8 ⇒ 428 +

𝑥 = 480 ⇒ 𝑥 = 480 − 428 = 52.

65. If the weight of a piece of diamond be 6x unit, then Original

price 6x 2 = 36kx2 , 36kx2 = 5184

Again New price = k(x2 + 4x2 + 9x2) = 14kx2 =14×5184

36=

Rs. 2016. Loss = 5184 − 2016 = Rs. 3168.

66. 30%=Rs. 30, 100% = Rs. 100, New S.P = 100-30= Rs. 70.

67. Part of the tank filled by both pipes in two hours =

2(1

8+

1

6) = 2

3+4

24 =

7

12 , Remaining part = 1 −

7

12=

5

12 .Time taken by B in filling the remaining part =

5

12× 6 =

5

2= 2

1

2hours.

68. Interest = 5700-5000=Rs. 700. Rate = 700×100

5000 ×1= 14%

Case II, Interest = Principal ×Time ×Rate

100=

7000 ×5×14

100=

Rs. 4900. Amount = 7000 + 4900 = Rs. 11900.

69. 75070÷65, R=60, Required number = 75070+(65-60)=75070.

70. According to the question, n

2+

n

4+

n

5+ 7 = n ⇒

10n+5n+4n

20+ 7 = n ⇒

19n

20+ 7 = n ⇒

n −19n

20= 7 ⇒

n

20= 7 ⇒ n = 20 × 7 = 140.

71. Required percentage increases = 40−30

30× 100 =

100

3=

331

3%.

72. Income of company in 2002 = Rs. 40 lakh

Expenditure of company in 2003 = Rs. 40 lakh.

73. Profit of company in 2004 = Rs. 25 lakh.

74. Required difference = 20-10= Rs. 10 lakh.

75. Average income of company = 30+50+40+60+60

5=

240

5=

Rs. 48 lakh. The incomes of company in years 2001, 2003 and

2004 were greater than Rs, 48 lakh.

76. Here, look like …..should be used. Like is an Adjective which is

used as Preposition.

77. Here, foot tall …. should be used.

78. Here, with no proof of your guilt/without any proof of your

guilt ….. should be used.

79. Here, she appears to be unreasonably (Adverb) …. should be

used because an adverb modifies an Adjective.

80. Here, the scissors are …. should be used.

Look at the sentences: My scissors are sharp.

A pair of scissors has been purchased.

Look at some inseparables: scissors, Shears, trousers.

81. The word Nexus (Noun) means: a complicated series of

connections.

Look at the sentence: There is a strong nexus between a

criminal and corrupt police.

82. The word Hyperbole (Noun) means: a way of speaking or

writing that makes something sound better, more exciting

than it really is; exaggeration. Hence, its antonym should be

faulty.

83. The word Amalgamate (Verb) means: merge; to put two or

more things together so that they form one; assimilate.

Look at the sentence: the company has now amalgamated

with another local firm.

Its antonym should be separate which means: to divide into

different parts or groups: to move apart.

84. The word Influx (Noun) means: the fact of a lot of people,

money or things arriving somewhere. The word Exodus

(Noun) means: a situation in which many people leave a place

at the same time.

85. Idiom have an axe to grind means: to have private reasons

for being involved in something or for arguing for a particular

cause.

86. Idiom put your foot down means: to be very strict in

opposing what somebody wishes to do , to drive faster.

87. Idiom Gainsay (verb) means: to disagree; to deny.

PRACTICE SET-3 General Intelligence & Reasoning

1. A group of friends are sitting in an arrangement one each

at the corner of an octagon. All are facing the centre. Mahima is sitting diagonally opposite Rama, who is on Sushma’s right. Ravi is next to Sushma and opposite Girdhar, who is on Chandra’s left. Savitri is not on Mahima’s right but opposite Shalini. Who is on Shalini’s right? (1) Ravi (2) Mahima (3) Girdhar (4) Rama

Directions (2-6) : In each of the following questions, select the related word/letters/number from the given alternatives. 2. Fox : Cunning : : Rabbit : ?

(1) Courageous (2) Dangerous (3) Timid (4) Ferocious

3. AZCX : BYDW : : HQJO : ? (1) GRFP (2) IPKM (3) IPKN (4) GRJP

4. 𝐴𝐵𝐶

𝐹∶

𝐵𝐶𝐷

𝐼∶ :

𝐶𝐷𝐸

𝐿∶ ?

(1) 𝐷𝐸𝐹

𝑂 (2)

𝐷𝐸𝐹

𝑁

(3) 𝐸𝐷𝐹

𝑂 (4)

𝐷𝐸𝐹

𝑀

5. 24 : 126 : : 48 : ? (1) 433 (2) 192 (3) 240 (4) 344

6. 987 : IHG : : 654 : ? (1) FDE (2) FED (3) EFD (4) DEF

Directions (7-9) : In each of the following questions, find the odd word/letters/number from the given responses. 7. (1) Mouth Organ (2) Electric Guitar

(3) Keyboard (4) Sonata 8. (1) RNMP (2) JFEH

(3) RPOQ (4) HDCF 9. (1) 6243 (2) 2643

(3) 8465 (4) 4867 10. From amongst the given alternatives, select the one in

which the set of numbers is most like the set of numbers given below : (6, 14, 30)

TARGET NO-1 CLASSES (A UNIT OF) EXECUTIVE MAKERS PVT.LTD. V-157

FOR DOWNLOADING BOOKS : www.executivemakers.com Page-18

(1) 4, 16, 28 (2) 7, 12, 22 (3) 6, 12, 22 (4) 5, 12, 20

11. Arrange the following words according to the dictionary : (A) TORTOISE (B) TORONTO (C) TORPED (D) TORUS (E) TORSEL

(1) B, E, C, A, D (2) B, E, C, D, A (3) B, C, E, A, D (4) B, C, E, D, A

12. Which set of letters when sequentially placed at the gaps in the given letter series shall complete it?

_a_aaaba_ _ba_ab_ (1) abaaaa (2) abaaba (3) aababa (4) ababaa

Directions (13-15) : In each of the following questions, find the missing number/letters from the given responses. 13. a, r, c, s, e, t, g, _, _

(1) x, z (2) u, i (3) w, y (4) v, b

14. 0, 4, 18, 48, ?, 180 (1) 58 (2) 68 (3) 84 (4) 100

15. 36, 28, 24, 22, ? (1) 18 (2) 19 (3) 21 (4) 22

16. If the day after tomorrow is Sunday, what day was tomorrow’s day before yesterday?

(1) Friday (2) Thursday (3) Monday (4) Tuesday

17. X and Y are brothers. R is the father of Y. S is the brother of T and maternal uncle of X. What is T to R?

(1) Mother (2) Wife (3) Sister (4) Brother

18. A word/set of letters given in capital letters is followed by four answer words. Out of these only one cannot be formed by using the letters of the given word/set of letters. Find out the word :

INDETERMINATE (1) DETERMINE (2) RETINUE (3) REMINDER (4) RETINA

19. One statement is given followed by two assumptions, I and II. You have to consider the statement to be true, even if, it seems to be at variance from commonly known facts. You are to decide which of the given assumptions can definitely be drawn from the given statement. Indicate your answer. Statement : All the people of Kerala are literate. Assumption I : People of Kerala are well educated and cultured. Assumption II : People of Kerala are hard-working and sincere.

(1) Only I is implicit. (2) Only II is implicit. (3) Both I and II are implicit. (4) Neither I nor II is implicit.

20. If 841 = 3, 633 = 5, 425 = 7 then 217 = ? (1) 6 (2) 7 (3) 8 (4) 9

21. 25 * 2 * 6 = 4 * 11 * 0 Which set of symbols can replace * ?

(1) , -, , + (2) +, -, , +

(3) , +, , - (4) , +, +,

22. Find the missing number from the given responses.

5 6 12

4 3 4

2 3 ?

18 27 96

(1) 4 (2) 5 (3) 3 (4) 6

23. Peter walked 8 km West and turned right and walked 3 km. Then again he turned right and walked 12 km. How far is he from the starting point?

(1) 7 (2) 8 (3) 4 (4) 5

24. Choose the correct figure that represents the given relation : Blue-eyed, females, doctors

(1) (2)

(3) (4) 25. How many triangles are there in the following figure?

(1) 20 (2) 24 (3) 28 (4) 32

General Awareness

26. The first non-stop air-conditioned ‘DURANTO’ train was flagged off between

(1) Sealdah-New Delhi (2) Mumbai-Howrah (3) Bangalore-Howrah (4) Chennai-New Delhi

27. Which one of the following States does not form part of Narmada River basin?

(1) Madhya Pradesh (2) Rajasthan (3) Gujarat (4) Maharashtra

28. The exchange of commodities between two countries is referred as

(1) Balance of trade (2) Bilateral trade (3) Volume of trade (4) Multilateral trade

29. Soil erosion on hill slopes can be checked by (1) Afforestation (2) Terrace cultivation (3) Strip cropping (4) Contour ploughing

30. Which of the following is called the “ecological hot spot of India”?

(1) Western Ghats (2) Eastern Ghats (3) Western Himalayas (4) Eastern Himalayas

31. The monk who influenced Ashoka to embrace Buddhism was

(1) Vishnu Gupta (2) Upa Gupta (3) Brahma Gupta (4) Brihadratha

TARGET NO-1 CLASSES (A UNIT OF) EXECUTIVE MAKERS PVT.LTD. V-157

FOR DOWNLOADING BOOKS : www.executivemakers.com Page-19

32. The Lodi dynasty was founded by (1) Ibrahim Lodi (2) Sikandar Lodi (3) Bahlol Lodi (4) Khizr Khan

33. Which Governor-General is associated with Doctrine of Lapse?

(1) Lord Ripon (2) Lord Dalhousie (3) Lord Bentinck (4) Lord Curzon

34. Marx belonged to (1) Germany (2) Holland (3) France (4) Britain

35. The Speaker of the Lok-Sabha has to address his/her letter of resignation to

(1) Prime Minister of India (2) President of India (3) Deputy Speaker of Lok Sabha (4) Minister of parliamentary Affairs

36. The terms “Micro Economics” and “Macro Economics” were coined by

(1) Alfred Marshall (2) Ragner Nurkse (3) Ragner Frisch (4) J.M. Keynes

37. “Economics is what it ought to be”-This statement refers to

(1) Normative economics (2) Positive economics (3) Monetary economics (4) Fiscal economics

38. Tetra ethyle lead (TEL) is (1) a catalyst in burning fossil fuel (2) an antioxidant (3) a reductant (4) an antiknock compound

39. Curie point is the temperature at which (1) matter becomes radioactive (2) a metal loses magnetic properties (3) a metal loses conductivity (4) transmutation of metal occurs

40. Which of the following is not a nucleon? (1) proton (2) Neutron (3) Electron (4) Positron

41. The material used in the manufacture of lead pencil is (1) Graphite (2) Lead (3) Carbon (4) Mica

42. Processor’s speed of a computer is measured in (1) BPS (2) MIPS (3) Baud (4) Hertz

43. ‘C’ language is a (1) low level language (2) high level language (3) machine level language (4) assembly level language

44. If all bullets could not be removed from gun shot injury of a man, it may cause poisoning by

(1) Mercury (2) Lead (3) Iron (4) Arsenic

45. Pituitary gland is situated in (1) the base of the heart (2) the base of the brain (3) the neck (4) the abdomen

46. Who discovered cement?

(1) Agassit (2) Albertus Magnus (3) Joseph Aspdin (4) Janseen

47. TRIPS and TRIMS are the terms associated with (1) IMF (2) WTO (3) IBRD (4) IDA

48. A Presidential Ordinance can remain in force (1) for three months (2) for six months (3) for nine months (4) indefinitely

49. NIS stands for (1) National Infectious diseases Seminar (2) National Irrigation Schedule (3) National Immunisation Schedule (4) National Information Sector

50. Harshvardhana was defeated by (1) Prabhakaravardhana (2) Pulakesin II (3) Narasimhasvarma Pallava (4) Sasanka

Quantitative Aptitude

51. In how many years will a sum of Rs. 800 at 10% per annum compound interest, compounded semi-annually becomes Rs. 926.10?

(1) 11

2 (2) 1

2

3

(3) 21

3 (4) 2

1

2

52. In a 10 m race, Kamal defeats Bimal by 5 seconds. If the speed of Kamal is 18 kmph, then the speed of Bimal is

(1) 15.4 kmph (2) 14.5 kmph (3) 14.4 kmph (4) 14 kmph

53. A boatman rows 1 km in 5 minutes along the stream and 6 km in 1 hour against the stream. The speed of the stream is

(1) 3 kmph (2) 6 kmph (3) 10 kmph (4) 12 kmph

54. A can complete 1

3 of a work in 5 days and B,

2

5 of the work

in 10 days. In how many days both A and B together can complete the work?

(1) 10 (2) 93

8

(3) 84

5 (4) 7

1

2

55. One pipe fills a water tank three times faster than another pipe. If the two pipes together can fill the empty tank in 36 minutes, then how much time will the slower pipe alone take to fill the tank?

(1) 1 hour 21 minutes (2) 1 hour 48 minutes (3) 2 hours (4) 2 hour 24 minutes

56. The average of odd numbers upto 100 is (1) 50.5 (2) 50 (3) 49.5 (4) 49

57. (12 + 2

2 + 3

2 + ... + 10

2) is equal to

(1) 380 (2) 385 (3) 390 (4) 392

58. 1 −1

3 1 −

1

4 1 −

1

5 … . 1 −

1

25 is equal to

(1) 2

25 (2)

1

25

(3) 119

25 (4)

1

325

TARGET NO-1 CLASSES (A UNIT OF) EXECUTIVE MAKERS PVT.LTD. V-157

FOR DOWNLOADING BOOKS : www.executivemakers.com Page-20

59. A number, when divided by 136, leaves remainder 36. If the same number is divided by 17, the remainder will be

(1) 9 (2) 7 (3) 3 (4) 2

60. If 2p + 1

𝑃 = 4 the value of p

3 +

1

8𝑃3 is

(1) 4 (2) 5 (3) 8 (4) 15

61. 3.25 ×3.20−3.20 ×3.05

0.064 is equal to

(1) 1 (2) 1

2

(3) 1

10 (4) 10

62. The H.C.F and L.C.M of two numbers are 12 and 336 respectively. If one of the numbers is 84, the other is

(1) 36 (2) 48 (3) 72 (4) 96

63. If ‘n’ be any natural number, then by which largest number (n

3 – n) is always divisible?

(1) 3 (2) 6 (3) 12 (4) 18

64. If there is a profit of 20% on the cost price of an article, the percentage of profit calculated on its selling price will be

(1) 24 (2) 162

3

(3) 81

3 (4) 20

65. If the cost price of 15 books is equal to the selling price of 20 books, the loss per cent is

(1) 16 (2) 20 (3) 24 (4) 25

66. If on marked price, the difference of selling price with a discount of 30% and two successive discounts of 20% and 10% is Rs. 72, then the marked price (in rupees) is

(1) 3,600 (2) 3,000 (3) 2,500 (4) 2,400

67. Successive discounts of 10%, 20% and 30% is equivalent to a single discount of

(1) 60% (2) 49.6% (3) 40.5% (4) 36%

68. If each side of a square is increased by 10%, Its area will be increased by

(1) 10% (2) 21% (3) 44% (4) 100%

69. The ratio of income and expenditure of a person is 11 : 10. If he saves Rs. 9,000 per annum, his monthly income is

(1) Rs. 8,000 (2) Rs. 8,800 (3) Rs. 8,500 (4) Rs. 8,250

70. If W1 : W2 = 2 : 3 and W1 : W3 = 1 : 2 then W2 : W3 is (1) 3 : 4 (2) 4 : 3 (3) 2 : 3 (4) 4 : 5

71. A copper wire of length 36 m and diameter 2 mm is melted to form a sphere. The radius of the sphere (in cm) is

(1) 2.5 (2) 3 (3) 3.5 (4) 4

72. The ratio of the radii of two wheels is 3 : 4. The ratio of their circumferences is

(1) 4 : 3 (2) 3 : 4 (3) 2 : 3 (4) 3 : 2

Directions (73-75) : The pie-chart, given here, shows the amount of money spent on various sports by a school administration in a particular year .

73. If the money spent on Football was Rs. 9,000, how much

more money was spent on Hockey than on Football? (1) Rs. 11,000 (2) Rs. 11,500 (3) Rs. 12,000 (4) Rs. 12,500

74. If the money spent on Football was Rs. 9,000, what amount was spent on Cricket?

(1) Rs. 31,000 (2) Rs. 31,500 (3) Rs. 32,000 (4) Rs. 32,500

75. If the money spent on Football is Rs. 9,000 then what was the total amount spent on all sports?

(1) Rs. 73,000 (2) Rs. 72,800 (3) Rs. 72,500 (4) Rs. 72,000

English Language

Directions (76-80) : In the following questions, a part of the sentence is printed in bold. Below are given alternatives to the bold part at (1), (2) and (3) which may improve the sentence. Choose the correct alternative. In case no improvement is needed, your answer is (4). 76. To get into the building I’ll disguise as a reporter.

(1) disguise to be (2) disguise as one (3) disguise myself (4) No improvement

77. He denied that he had not forged my signature. (1) would not forge (2) had forged (3) did not forge (4) No improvement

78. If I had played well, I would have won the match. (1) I played well (2) I play well (3) I am playing well (4) No improvement

79. Since the records are missing, the possibility of paying more than one compensation for the same piece of land cannot be ruled aside.

(1) out (2) off (3) away (4) No improvement

80. A callous system generates nothing but a misanthrope. (1) develops (2) induces (3) produces (4) No improvement

Directions (81-82) : In the following questions, choose the word opposite n meaning to the given word. 81. JETTISON

(1) accept (2) reward (3) preserve (4) consent

82. DEVIOUS (1) straight (2) obvious (3) simple (4) superficial

TARGET NO-1 CLASSES (A UNIT OF) EXECUTIVE MAKERS PVT.LTD. V-157

FOR DOWNLOADING BOOKS : www.executivemakers.com Page-21

Directions (83-84) : In the following questions, out of the four alternatives, choose the one which best expresses the meaning of the given word. 83. OSTRACISE

(1) banish (2) belittle (3) beguile (4) besiege

84. CODDLE (1) huddle (2) satisfy (3) protect (4) cheat

Directions (85-86) : In the following questions, out of the four alternatives, choose the one which can be substituted for the given words/sentence. 85. One who hides away on a ship to obtain a free passage.

(1) Compositor (2) Stoker (3) Stowaway (4) Shipwright

86. An emolument over and above fixed income or salary. (1) Honorarium (2) Sinecure (3) Perquisite (4) Prerogative

Directions (87-88) : In the following questions, groups of four words are given. In each group, one word is correctly spelt. Find the correctly spelt word. 87. (1) parapharnelia (2) parasimonious

(3) peccadilo (4) peadiatrics 88. (1) tussel (2) tunnle

(3) tumble (4) trable Directions (89-90) : In the following questions, sentences are given with blanks to be filled in with and appropriate words(s). Four alternatives are suggested for each question. Choose the correct alternative out of the four. 89. The court .... cognisance of the criminal’s words.

(1) took (2) made (3) gave (4) allowed

90. Everyone in this world is accountable to God .... his actions.

(1) about (2) for (3) to (4) over

Directions (91-95) : In the following questions, you have one brief passage with 5 questions following the passage. Read the passage carefully and choose the best answer to each question out of the four alternatives. In May 1966, the World health Organization was authorised to initiate a global campaign to eradicate small-pox. The goal was to eradicate the disease in one decade. Because similar projects for malaria and yellow fever had failed, few believed that smallpox could actually be eradicated, but eleven years after the initial organization of the campaign, no cases were reported in the field. The strategy was not only to provide mass vaccinations, but also to isolate patients with active small-pox in order to contain the spread of the disease and to break the chain of human transmission. Rewards for reporting small-pox assisted in motivating the public to aid health workers. One by one, each small-pox victim was sought out, removed from contact with others and treated. At the same time, the entire, village where the victim had lived was vaccinated. Today small-pox is no longer a threat to humanity. Routine vaccinations have been stopped worldwide. 91. Which of the following is the best title for the passage?

(1) The World Health Organization

(2) The Eradication of Small-pox (3) Small-pox Vaccinations (4) Infectious Diseases

92. What was the goal of the campaign against small-pox? (1) To decrease the spread of small-pox worldwide. (2) To eliminate small-pox world-wide in ten years. (3) To provide mass vaccinations against small-pox worldwide. (4) To initiate worldwide projects for small-pox, malaria and yellow fever at the same time.

93. According to the paragraph, what was the strategy used to eliminate the spread of small-pox?

(1) Vaccination of the entire village (2) Treatment of individual victims (3) Isolation of victims and mass vaccinations (4) Extensive reporting of outbreaks

94. Which statement doesn’t refer to small-pox? (1) Previous projects had failed. (2) People are no longer vaccinated for it. (3) The World health Organization mounted a worldwide campaign to eradicate the disease. (4) It was a serious threat.

95. It can be inferred that (1) no new cases of small-pox have been reported this year. (2) malaria and yellow fever have been eliminated. (3) small-pox victims no longer die when they contract the disease. (4) small-pox is not transmitted from one person to another.

Directions (96-100) : Some of the sentences have errors and some have none. Find out which part of a sentence has an error. The number of that part is your answer. If there is no error, your answer is (4). 96. Judge in him (1) / prevailed upon the father (2) / and he

sentenced his son to death. (3) / No error (4) 97. Nine-tenths (1) / of the pillar (2) / have rotted away. (3) /

No error (4) 98. One major reason (1) / for the popularity of television is

(2) / that most people like to stay at home. (3) / No error (4)

99. Our efforts are (1) / aimed to bring about (2) / reconciliation. (3) / No error (4)

100. Three conditions (1) / critical for growing (2) / plants are soil, temperature, chemical balance or amount of moisture. (3) / No error (4)

TARGET NO-1 CLASSES (A UNIT OF) EXECUTIVE MAKERS PVT.LTD. V-157

FOR DOWNLOADING BOOKS : www.executivemakers.com Page-22

ANSWER SHEET

1.1 2.3 3.3 4.1 5.4 6.2 7.4

8.3 9.4 10.2 11.3 12.1 13.2 14.4

15.3 16.2 17.2 18.2 19.4 20.4 21.1

22.4 23.4 24.4 25.3 26.1 27.2 28.3

29.2 30.1 31.2 32.3 33.2 34.1 35.3

36.4 37.1 38.4 39.2 40.4 41.1 42.2

43.2 44.4 45.2 46.3 47.2 48.2 49.4

50.2 51.1 52.3 53.1 54.2 55.4 56.2

57.2 58.1 59.4 60.2 61.4 62.2 63.2

64.2 65.4 66.1 67.2 68.2 69.4 70.1

71.2 72.2 73.1 74.3 75.4 76.3 77.2

78.4 79.1 80.3 81.1 82.1 83.1 84.2

85.3 86.1 87.2 88.3 89.1 90.2 91.2

92.2 93.3 94.1 95.3 96.1 97.2 98.3

99.2 100.3

Practice Set – 3

Hints & Solution 1.

Ravi is to the right of Shalini.

2. Here, animal – behaviour relationship has been shown. Fox is

characterized by its cunningness. Similarly, rabbit is

considered as timid.

3. A(+1)→B, Z(-1)→Y, C(+1)→D, X(-1)→W Similarly H(+1)→I,

Q(-1)→P, J(+1)→K, O(-1)→N.

4. ABC(F)(+3)→BCD(I)::CDE(L)(+3)→DEF(O).

5. 52 − 1 = 24; 53 + 1 = 126; 72 − 1 = 48; 73 + 1 = 344.

6. 987 (IHG), 654 (FED)

7. Except Sonata, all other are instruments. Sonata is a piece of

music composed for one instrument or two.

8. R(-4)→N(-1)→M(+3)→P,

J(-4)→F(-1)→E(+3)→H,

R(-2)→P(-1)→O(+2)→Q,

H(-4)→D(-1)→C(+3)→F,

9. 6 + 2 = 8; 4 + 3 = 7 ⇒ 8 − 7 = 1

2 + 6 = 8; 4 + 3 = 7 ⇒ 8 − 7 = 1

8 + 4 = 12; 6 + 5 = 11 ⇒ 12 − 11 = 1

4 + 8 = 12; 6 + 7 = 13 ⇒ 12 − 13 = −1

10. 6 + 8 = 14; 14 + 8 × 2 = 30

7 + 5 = 12; 12 + 5 × 2 = 22

11. Arrangement of words according to Dictionary:

(B)Toronto→(C)Torped→(E)Torsel→(A)Tortoise

→(D)Torus

12. a a b a / a a b a / a a b a / a a b a .

13. There are two alternating series.

a(+2)→c(+2)→e(+2)→g(+2)→i

r(+1)→s(+1)→t(+1)→u, Therefore, ? =ui.

14. +4,+14,+30,+52,+80

+10,+16,+22,+28

+6,+6,+6

15. 36(-8)→28(-4)→24(-2)→22(-1)→21.

16. The day after tomorrow is Sunday. Therefore, today is Friday.

The day on tomorrow’s day before yesterday = Friday -1

=Thursday.

17. R is father of X and Y. S is maternal uncle of X and Y.

considering the given options, it may be assumed that T is

Wife of R.

18. There is no ‘U’ letter in the given word.

19. Neither I nor II is implicit in the statement. Literate has very

general implication. So, it is not necessary that people of

Kerala are well educated and cultured.

20. 8

4= 2; 2 + 1 = 3,

6

3= 2; 2 + 3 = 5,

4

2= 2; 2 + 7 = 9.

21. 25 × 2 − 6 = 4 × 11 + 0 ⇒ 50 − 6 = 44 + 0 ⇒ 44 = 44.

22. 5 + 4 = 9 and 9 × 2 = 18, 6 + 3 = 9 and 9 × 3 = 27, 12 +

4 = 16 and ? = 96

16= 6.

23.

AE = (AD)2 + (DE)2 = (3)2 + (4)2 = 9 + 16 = 25 =

5 km.

24. Some blue-eyed may be females and vice-versa. Some

females may be doctors and vice-versa. Some blue-eyed may

be doctors and vice-versa. Some blue-eyed females may be

doctors.

25. The triangles are :

∆FEB ; ∆CBD; ∆FAG; ∆ FEG; ∆BCI; ∆CDI; ∆AFI; ∆EFI; ∆BGC;

∆DCG; ∆AGI; ∆BIH; ∆AGB; ∆ABI; ∆HAB; ∆HBI; ∆HGI;

∆HAG; ∆GEI; ∆GED; ∆IDE; ∆IDG; ∆JGI; ∆JDI; ∆ JGE; ∆JDE;

∆AIE; ∆BGD. (28 Triangles)

26. Sealdah-New Delhi

27. Rajasthan

28. Volume of trade

TARGET NO-1 CLASSES (A UNIT OF) EXECUTIVE MAKERS PVT.LTD. V-157

FOR DOWNLOADING BOOKS : www.executivemakers.com Page-23

29. Terrace cultivation

30. Western Ghats

31. Upa Gupta

32. Bahlol Lodi

33. Lord Dalhousie

34. Germany

35. Deputy Speaker of Lok Sabha

36. J.M. Keynes

37. Normative economics

38. an antiknock compound

39. A metal loses magnetic properties

40. Positron

41. Graphite

42. MIPS

43. High level language

44. Arsenic

45. The base of the brain

46. Joseph Aspdin

47. WTO

48. For six months.

49. National Information Sector

50. Pulakesin II

51.

52. Time taken by Kamal = 100

18×5

18

= 20 seconds

∴Time taken by Bimal = 20+5=25 seconds.

∴Bimal’s speed = 100

25= 4m/ sec =

4×18

5kmph = 14.4 kmph.

53.

54. Time taken by A Alone in doing the work = 15 days

Time taken by B alone in doing the work = 10×5

2=

25 days.∴ A + B ′s1 day′s work =1

15+

1

25=

5+3

75=

8

75

∴Hence, the work will be completed in 75

8= 9

3

8days.

55.

56.

57.

58.

59.

60. 2𝑝+1

𝑝= 4 ⇒ 𝑝+

1

2𝑝= 2, ∴ (𝑝+

1

2𝑝)3 = 𝑝3 +

1

8𝑝3 +

3. 𝑝.1

2𝑝 𝑝+

1

2𝑝 ⇒ 8 = 𝑝3 +

1

8𝑝3 +3

2× 2 ⇒ 𝑝3 +

1

8𝑝3 = 8 − 3 =

5

61. Expression = 3.20 (3.25−3.05)

0.064=

3.02×0.20

0.064= 10.

62. First number x Second number = HCF × LCM

84× second number = 12×336

∴Second number = 12×336

84= 48.

63. n3 − n = n n2 − 1 = n n + 1 n − 1

For n = 2, n3 − n = 6.

64.

65. If the cost price of each book be Rs. 1, then SP of 20 books =

Rs. 15. CP of 20 books = Rs. 20

Trick

Rate = 10% per annum; = 5% half yearly

𝐴 = 𝑃 1 +𝑅

100 𝑇

⇒ 926.10 = 800 1 +5

100 𝑇

⇒9261

8000=

21

20 𝑇

21

20

3

= 21

20

T

∴ Time = 3 half years = 11

2years.

[Rate downstream =1

5× 60 = 12 kmph]

Trick

Speed of current = 1

2 Rate downstream −

Rate upsream =1

2 12 − 6 kmph =

3 kmph

Trick If time taken by the pipe at faster rate to fill the tank

be x minutes, then = 1

x+

1

3x=

1

36⇒

3+1

3x=

1

36⇒ 3x =

4 × 36 ⇒ x = 48 minutes. Time taken by the slower pipe = 48 × 3 = 144 minutes =2

hours 24 minutes.

Trick

Average of the first n natural odd numbers = n

Number of odd numbers upto 100 = 50= required

average.

Trick

12 + 22 + 32 + 42+. . . +n2 =n n+1 (2n+1)

6, ∴ 12 +

22 + 32 + 42+. . . +102 =10 10+1 (20+1)

6= 385

Trick

1 −1

3 1 −

1

4 1 −

1

5 … 1 −

1

24 1 −

1

25 =

2

3

4

5. .×

23

24×

24

25=

2

25.

Trick

If the first divisor be a multiple of the second divisor,

then required remainder= remainder obtained by

dividing the first remainder (36) by the second

divisor (17)=2.

Trick

If the cost price = Rs. 100 then selling price =

Rs. 120 and gain = Rs. 20 Required gain %= 20

120× 100

TARGET NO-1 CLASSES (A UNIT OF) EXECUTIVE MAKERS PVT.LTD. V-157

FOR DOWNLOADING BOOKS : www.executivemakers.com Page-24

∴Loss percent = 20−15

20× 100 = 25%.

66. Let the marked price be Rs. x. In case I,

∴SP = Rs. 70x

100 .Single discount equivalent to successive

discounts of 20% and 10%. = 20 + 10 −20×10

100 % = 28%

∴S.P. in this case = Rs. 72x

100,

∴ 72x

100−

70x

100= Rs. 72 ⇒

2x

100= 72 ⇒ x =

72×100

2= Rs. 3600.

67.

68.

69. Let the income of man be Rs. 11x and his expenditure be Rs.

10x. ∴Savings = x =Rs. 9000.

∴Monthly income of man. = 11×9000

12= Rs. 8250.

70.

71. Volume of the wire =𝜋r2h = π × 0.1 × 0.1 × 3600 cm3 =

36πcm3 Volume of the sphere = 4

3πR3 = 36π ⇒ R3 =

36×3

4=

27,∴ R = 273

= 3 cm.

72.

73. ∴45°=Rs. 9000 ∴55°=9000

45°× 55° = Rs. 11000.

74. ∴45°=Rs. 9000 ∴160°=9000

45°× 160° = Rs. 32000.

75. ∴45°=Rs. 9000 ∴360°=9000

45°× 360° = Rs. 72000.

76. disguise myself

77. had forged

78. No improvement

79. out

80. Produces

81. The word Jettison (Verb) means: to throw something;

abandon; to reject an idea. Hence, the words Jettison and

accept are antonymous.

82. The word Devious (Adjective) means: behaving in a dishonest

way: a route that is not straight.

Hence, the words devious and straight are antonymous.

83. The word Ostracise (Verb) means: to refuse to let somebody

be a member of a social group; refuse; shun. Look at the

sentence: He was ostracized by his colleagues for refusing to

support the strike. Hence, the words banish and ostracise are

synonymous.

84. The word Coddle (Verb) mean: to treat somebody with too

much care and attention, pamper, cosset. Hence, the words

coddle and satisfy are synonymous.

85. Stowaway

86. Honorarium

87. Correct spellings of other words are: paraphernalia.

Peccadillo and paediatrics.

88. Correct spellings of other words are: tussle, tunnel and treble.

89. Idiom take cognizance of something means: to understand

or consider something; to take notice of something.

90. for

PRACTICE SET-4 General Intelligence & Reasoning

1. Which one is different from the rest three?

(1) Polar Bear (2) Penguin (3) Seal (4) Tiger

2. Which one set of letters when sequentially placed at the gaps in the given letter series shall complete it?

_ab_b_bc_ca_ (1) accbb (2) abcca (3) cacab (4) abacb

Directions (3-5) : In each of the following questions, choose the correct alternative from the given ones that will complete the series. 3. (1) A, C, E (2) B, D, F

(3) G, I, K (4) ? (1) L, J, H (2) Z, Y, X (3) H, J, L (4) K, L, M

4. 30, 24, 19, 15, 12, ? (1) 6 (2) 8 (3) 10 (4) 11

5. 30, 28, 23, 21, ? (1) 15 (2) 16 (3) 18 (4) 20

6. If 10 boys build 10 km road in 10 days, then how many days it will take for 3 boys to build 10 km road? (1) 10 (2) 3/10 (3) 6/8 (4) 100/3

7. From the given alternatives, select the word which cannot be formed using the letters of the given word:

FRAGMENT (1) RAGE (2) TEAR (3) MEAN (4) RACE

8. If MARCH is coded as PXUZK what will be the code of APRIL?

Trick

Single equivalent discount for successive discounts

of 10% and 20%. = 10 + 20 −10×20

100 % =

28%. Single equivalent discount for 28% and 30%.

= 28 + 30 −28×30

100 % = 49.6%.

Trick

Increase percent in area = (10 + 10 +10×10

100)% =

21%.

Trick W1

W2=

2

3⇒

W2

W1=

3

2 and

W1

W3=

1

2 ,

∴ W2

W1×

W1

W3=

W2

W3=

3

1

2=

3

4.

Trick

Ratio of the circumferences = Ratio of radii = 3:4.

TARGET NO-1 CLASSES (A UNIT OF) EXECUTIVE MAKERS PVT.LTD. V-157

FOR DOWNLOADING BOOKS : www.executivemakers.com Page-25

(1) DMUFO (2) DSULO (3) ZKIRO (4) ZKRIO

9. If A denotes +, B denotes – and C denotes , then (10 C 4) A (4 C 4) B 6 = ? (1) 46 (2) 50 (3) 55 (4) 58

Directions (10-12) : In each of the following equations, select the missing number from the given responses. 10.

5 4 3 8 9 4

20 9 24 11 ? 13

(1) 36 (2) 117 (3) 52 (4) 26

11.

9 30 21

6 ? 14

12 40 28

(1) 20 (2) 33 (3) 37 (4) 70

12.

28 4 25 5 ? 3

7 11 5 10 8 11

(1) 22 (2) 24 (3) 25 (4) 28

13. A student walked out from the Classroom towards the Library. She went first to the Canteen on the north side, 24 ft away. After a cup of tea, took a right turn and went to the Laboratory 13 ft away. She then went to the Physics Block, 15 ft to the left. She talked to a friend, in the garden 3 ft on the left and continued walking in the same direction, to the Library, 10 ft more. What was the actual distance between the Library and the Classroom?

(1) 42 ft (2) 65 ft (3) 39 ft (4) 34 ft

14. A husband and a wife had five married sons and each of them had four children. How many members are there in the family?

(1) 32 (2) 36 (3) 30 (4) 40

Directions (15-17) : In each of the following questions, select the related letter/word/number from the given alternatives. 15. India : Mango : : New Zealand : ?

(1) Apples (2) Kiwi (3) Grapes (4) Bananas

16. VZS : EAH : : ? : JFM (1) QSM (2) NUQ (3) QUN (4) QNU

17. 42 : 20 : : 64 : ? (1) 31 (2) 32

(3) 40 (4) 42 Directions (18-20) : In each of the following questions, find the odd number/letters/number pair from the given alternatives. 18. (1) Orange (2) Apple

(3) Lemon (4) Grapes 19. (1) 725 (2) 840

(3) 632 (4) 475 20. (1) 64 - 36 (2) 75 - 35

(3) 57 - 43 (4) 39 - 61 21. Which one of the given responses would be a meaningful

order of the following words? (1) Type (2) Print (3) Open (4) Save (5) Close (1) 3, 4, 1, 2, 5 (2) 3, 5, 4, 2, 1 (3) 3, 1, 4, 2, 5 (4) 3, 2, 1, 4, 5

22. How many rectangles are there in the given figure?

(1) 6 (2) 4 (3) 8 (4) 10

Directions (23) : In the following question, identify the diagram that best represents the relationship among classes given below. 23. Government Servants, Lecturers, Doctors

(1) (2)

(3) (4) 24. A piece of paper is folded and cut as shown below in the

question figures. From the given answer figures, indicate how it will appear when opened. Questions Figures :

Answers Figures :

(1) (2)

(3) (4) 25. Identify the alternative which resembles the mirror-

image of the given word. SECRETARY

(1) (2)

(3) (4)

TARGET NO-1 CLASSES (A UNIT OF) EXECUTIVE MAKERS PVT.LTD. V-157

FOR DOWNLOADING BOOKS : www.executivemakers.com Page-26

English Language

Directions (26-30) : In the following questions, some parts of the sentences have error and some are correct. Find out which part of a sentence has an error. The number of that part is your answer. If a sentence is free from error, your answer is (4) i.e., No error 26. The United Nations (1) / enquired for (2) / a cease fire.

(3) / No error (4) 27. Ghana, Nigeria and Gambia (1) / each have parts to play

(2) / in the development of Africa. (3) / No error (4) 28. In these days of inflation (1) / a ten rupees note will not

buy you (2) / even an ordinary meal. (3) / No error (4) 29. Most of my friends (1) / heard the earthquake (2) / but I

was totally unaware of it. (3) / No error (4) 30. He walks (1) / as if the earth (2) / belonged to him (3) /

No error (4) Directions (31-33) : In the following questions, four alternatives are given for the Idiom/Phrase printed in bold in the sentence. Choose the alternative which best expresses the meaning of the Idiom/Phrase. 31. The departmental store is open around the clock.

(1) at different timings (2) early morning (3) day and night (4) throughout the afternoon

32. Through the reporter’s efforts, many unknown facts have come to light.

(1) ignited (2) flared up (3) brightened (4) been revealed

33. The boy was in Dutch with his friends. (1) in love (2) in good terms (3) in awe (4) in trouble

Directions (34-36) : In the following questions, out of the four alternatives, choose the one which can be substituted for the given words/sentence. 34. A person who drives our motor cars.

(1) Driver (2) Chauffeur (3) Conductor (4) Lift-operator

35. Short speech or poem given at the end of a play or a book.

(1) Epilogue (2) Epigram (3) Epitaph (4) Epicure

36. Lack of feeling (1) Empathy (2) Apathy (3) Sympathy (4) Pity

Directions (37-38) : In the following questions, out of the four alternatives, choose the one which best expresses the meaning of the given word as your answer. 37. Jealous

(1) Interested (2) Hatred (3) Envied (4) Admired

38. Futility (1) Uselessness (2) Insignificance (3) Irrelevance (4) Unimportance

Directions (39-40) : In the following questions, choose the word opposite in meaning to the given word as your answer. 39. Prudent

(1) Dunce (2) Silly (3) Foolish (4) Careless

40. Turbulent (1) Harmony (2) Gusty (3) Calm (4) Windy

Directions (41-50) : In the following questions, in the following passage some of the words have been left out. Read the passage carefully and choose the correct answer to each question out of the four alternatives and fill in the blanks. The order day I visited a refugee ...(41)... where the victims ...(42)... the Gujarat. Earthquake ...(43)... in very ...(44)... conditions. I was particularly ...(45)... by an old woman who was determined to give ...(46)... grand-children a better future. She ...(47)... a strong and ...(48)... woman who even after the ...(49)... of her own children undertook such a journey through life and never felt weak or broken but was an ...(50)... for all. 41. (1) house (2) camp

(3) home (4) nest 42. (1) of (2) to

(3) in (4) at 43. (1) was living (2) are living

(3) were living (4) have lived 44. (1) apathetic (2) sympathetic

(3) pathetic (4) empathetic 45. (1) cornered (2) collected

(3) worked (4) moved 46. (1) his (2) her

(3) its (4) their 47. (1) were (2) had

(3) was (4) is 48. (1) courageous (2) continuous

(3) ruinous (4) careful 49. (1) life (2) death

(3) motionless (4) captivated 50. (1) happiness (2) determination

(3) motivation (4) inspiration

Quantitative Aptitude

51. A got twice as many marks in English as in Science. His total marks in English, Science and Mathematics is 180. If the ratio of his marks in English and Mathematics is 2 : 3, what is his marks in Science?

(1) 20 (2) 60 (3) 30 (4) 40

52. Three years ago, the average age of a family of 5 members was 17 years. A baby having been born, the average age of the family remains the same today. The age of the baby today is

(1) 3 years (2) 2 years (3) 1 years (4) 1.5 years

53. A shoe company sold 50 pairs of shoes on a day costing Rs. 189.50 each for Rs. 10,000. Then, the profit obtained in Rs. is

(1) 522 (2) 525 (3) 573 (4) 612

54. Subtracting x% of y from y means multiplying y by

(1) 𝑥

100 (2)

𝑥

100− 1

TARGET NO-1 CLASSES (A UNIT OF) EXECUTIVE MAKERS PVT.LTD. V-157

FOR DOWNLOADING BOOKS : www.executivemakers.com Page-27

(3) 1 −𝑥

100 (4) x 1 −

𝑥

100

55. In a village 30% of the population is literate. If the total population of the village is 6,600 then the number of illiterate is

(1) 1980 (2) 4620 (3) 2200 (4) 3280

56. Two trains start from stations A and B and travel towards each other at speeds of 50 kmph and 60 kmph respectively. At the time of their meeting, the second train has travelled 120 km more than the first. The distance between A and B is

(1) 1200 km (2) 1440 km (3) 1320 km (4) 990 km

57. Find the difference between the compound interest and the simple interest on Rs. 32,000 at 10% p.a. for 4 years.

(1) Rs. 2051.20 (2) Rs. 2052.50 (3) Rs. 2025.20 (4) Rs. 2501.20

58. If each edge of a square be doubled then the increase percentage in its area is

(1) 200% (2) 250% (3) 280% (4) 300%

59. A solid metallic spherical ball of diameter 6 cm is melted and recast into a cone with diameter of the base as 12 cm. The height of the cone is

(1) 2 cm (2) 3 cm (3) 4 cm (4) 6 cm

60. If a + b + c = 0, then the value of (a + b - c)2 + (b + c – a)

2 +

(c + a – b)2 is

(1) 0 (2) 8 abc (3) 4(a

2 + b

2 + c

2) (4) 4(ab + bc + ca)

61. If x = 2015, y = 2014 and z = 2013, then value of x2

+ y2 + z

2 – xy – yz - zx is

(1) 3 (2) 4 (3) 6 (4) 2

62. If x > 1 and x + 1

𝑥= 2

1

12, then the value of 𝑥4 −

1

𝑥4 is

(1) 58975

20736 (2)

59825

20736

(3) 578985

20736 (4)

57895

20736

63. AD is the median of a triangle ABC and O is the centroid such that AO = 10 cm. Length of OD (in cm) is

(1) 2 (2) 4 (3) 5 (4) 7

64. Two circles C1 and C2 touch each other internally at P. Two lines PCA and PDB meet the circles C1 in C, D and C2

in A, B respectively. If BDC = 1200, then the value of

ABP is equal to (1) 60

0 (2) 80

0

(3) 1000 (4) 120

0

65. The distance between the centres of two circles with radii 9 cm and 16 cm is 25 cm. The length of the segment of the tangent between them is

(1) 24 cm (2) 25 cm

(3) 50

3 cm (4) 12 cm

66. If sin ( + 180) = cos 60

0 (0 < < 90

0), then the value of

cos 5 is

(1) 1

2 (2) 0

(3) 1

2 (4) 1

67. If tan = 3

4 , then the value of

4 𝑠𝑖𝑛 2𝜃−2 𝑐𝑜𝑠 2𝜃

4 𝑠𝑖𝑛 2𝜃+3 𝑐𝑜𝑠 2𝜃 is equal to

(1) 1

21 (2)

2

21

(3) 4

21 (4)

8

21

68. Let A, B, C, D be the angles of a quadrilateral. If they are concyclic, then the value of

cos A + cos B + cos C + cos D is (1) 0 (2) 1 (3) -1 (4) 2

69. If x : y be the ratio of two whole numbers and z be their HCF, then the LCM of those two numbers is

(1) yz (2) 𝑥𝑧

𝑦

(3) 𝑥𝑦

𝑧 (4) xyz

70. If 12 men or 24 boys can do a work in 66 days, the number of days in which 15 men and 6 boys can do it, is

(1) 44 (2) 33 (3) 55 (4) 66

71. If diagonals of a rhombus are 24 cm and 32 cm, then perimeter of that rhombus is

(1) 80 cm (2) 84 cm (3) 76 cm (4) 72 cm

Directions (72-75) : The marks obtained by 273 examinees are shown by the frequency polygon, Given that mean marks is 59.5. Study the frequency polygon and answer the given questions.

72. The number of examinees getting more than average

marks is (1) 72 (2) 105 (3) 152 (4) 164

73. percentage of the students who get above 80% marks is (1) 9.81 (2) 10.53 (3) 11.28 (4) 11.72

74. Percentage of the students who got marks above 60% and below 80% is

(1) 43.95 (2) 48.39 (3) 51.06 (4) 56.84

75. If 40 is the pass marks, percentage of students failed is (1) 14.56 (2) 15.84 (3) 16.11 (4) 17.25

General Awareness

76. Which one of the following is the world’s largest desert? (1) Arabian (2) Kalahari (3) Sahara (4) Thar

77. The temperature increases rapidly above (1) Ionosphere (2) Exosphere (3) Stratosphere (4) Troposphere

TARGET NO-1 CLASSES (A UNIT OF) EXECUTIVE MAKERS PVT.LTD. V-157

FOR DOWNLOADING BOOKS : www.executivemakers.com Page-28

78. Which one of the following is not a photosynthetic pigment?

(1) Chlorophyll (2) Phycobilin (3) Carotenoid (4) Anthocyanin

79. Which of the following is a fibrous protein? (1) Haemoglobin (2) Albumin (3) Keratin (4) Enzymes

80. Carolus Linnaeus System of Classification is (1) Natural (2) Artificial (3) Binomial (4) Phylogenetic

81. The time between program input and output is called (1) Turn around time (2) Waiting time (3) Execution time (4) Delay time

82. The Indian National Grid Computing Initiative for Scientific Engineering and Academic Community is named

(1) Ganga (2) SAGA (3) Garuda (4) PARAM

83. Water has maximum density at (1) 100

0 C (2) 0

0 C

(3) 40

C (4) 2730

C 84. Human Rights Day is observed on

(1) 10th

March (2) 10th

July (3) 10

th September (4) 10

th December

85. Ajanta Caves were built during period of (1) Gupta (2) Kushana (3) Maurya (4) Chalukya

86. Who was the painter of the famous painting called ‘Bharat Mata’?

(1) Gaganendranath Tagore (2) Abanindranath Tagore (3) Nandalal Bose (4) Jamini Roy

87. “Take-off stage” in an economy means (1) Steady growth begins. (2) Economy is stagnant. (3) Economy is about to collapse. (4) All controls are removed.

88. The ‘Slack Season’ in the Indian Economy is (1) March-April (2) September-December (3) January-June (4) February-April

89. Which one of the Constitutional amendment has established Panchayati Raj Institution?

(1) 72nd

Amendment Act (2) 71

st Amendment Act

(3) 73rd

Amendment Act (4) 78

th Amendment Act

90. Which among the following States, first introduced the Panchayat Raj System?

(1) Rajasthan (2) Haryana (3) Uttar Pradesh (4) Maharashtra

91. Which Mughal Emperor transferred the Mughal Capital from Agra to Delhi?

(1) Jahangir (2) Aurangzeb (3) Shahjahan (4) Bahadur Shah

92. Who said, “A living thing is born” after the League of Nation’s Covenant was drafted?

(1) Lord Robert Cecil (2) Woodrow Wilson (3) Orlando (4) Neville Chamberlain

93. The author of ‘Arthashastra’ was a contemporary of

(1) Ashoka (2) Chandragupta Maurya (3) Samudragupta (4) Chandragupta Vikramaditya

94. The Taj-Mahal was built by (1) Jahangir (2) Shahjahan (3) Sher Shah (4) Nadir Shah

95. Which was the backbone of Indus Economy? (1) Agriculture (2) Trade (3) Wheel Made Pottery (4) Carpentry

96. Who discovered Cape of Good Hope in 1488? (1) Magellan (2) Columbus (3) Bartholomew Dias (4) Vasco da Gama

97. Sodium Carbonate is commonly known as (1) Baking Soda (2) Washing Soda (3) Caustic Soda (4) Caustic Potash

98. The Election commission is established under the (1) Article-355 (2) Article-256 (3) Article-324 (4) Article-320

99. Typhoid fever is caused by (1) Bacteria (2) Virus (3) Protozoa (4) Fungi

100. Which one of the following continents lies in Northern-Southern and Eastern-Western hemispheres of the earth?

(1) Australia (2) Africa (3) Europe (4) South America

ANSWER SHEET

1.4 2.3 3.3 4.3 5.2 6.4 7.4

8.1 9.2 10.1 11.1 12.2 13.3 14.1

15.2 16.3 17.4 18.2 19.3 20.1 21.3

22.3 23.4 24.4 25.4 26.2 27.2 28.2

29.2 30.4 31.3 32.4 33.4 34.2 35.1

36.2 37.3 38.1 39.4 40.3 41.2 42.1

43.3 44.3 45.1 46.2 47.3 48.1 49.2

50.4 51.3 52.2 53.2 54.3 55.2 56.3

57.1 58.4 59.2 60.3 61.1 62.1 63.3

64.1 65.1 66.1 67.1 68.1 69.3 70.1

71.1 72.3 73.4 74.1 75.3 76.3 77.2

78.4 79.3 80.3 81.1 82.3 83.3 84.4

85.1 86.2 87.1 88.3 89.3 90.1 91.3

92.2 93.2 94.2 95.1 96.3 97.2 98.3

99.1 100.2

TARGET NO-1 CLASSES (A UNIT OF) EXECUTIVE MAKERS PVT.LTD. V-157

FOR DOWNLOADING BOOKS : www.executivemakers.com Page-29

Practice Set -4

Hints & Solution 1. Except tiger, all others are region specific creatures.

2. c a b / a b c / b c a / c a b

3. A(+2)→C(+2)→E, B(+2)→D(+2)→F, G(+2)→I(+2)→K,

H(+2)→J(+2)→L.

4. 30(-6)→24(-5)→19(-4)→15(-3)→12(-2)→10.

5. 30(-2)→28(-5)→23(-2)→21(-5)→16.

6. 10 boys walk 10 km in 10 days, i.e., any number of boys can

walk 10 km in 10 days.

7. There is no ‘C’ letter in the given word. Therefore, the word

RACE cannot be formed.

8. M(+3)→P, A(-3)→X, R(+3)→U, C(-3)→Z, H(+3)→K

Therefore, A(+3)→D, P(-3)→M, R(+3)→U, I(-3)→F, L(+3)→O

9. A ⇒ +, B ⇒ −, C ⇒×

10C4 A 4C4 B6 =?

⇒? = 10 × 4 + 4 × 4 − 6

⇒? = 40 + 16 − 6 = 50.

10. The sum of upper two numbers gives the lower right number

while their product is equal to the lower left number.

First arrangement 5 + 4 = 9 and 5 × 4 = 20

Second arrangement 3 + 8 = 11 and 3 × 8 = 24

Third arrangement 9 + 4 = 13 and 9 × 4 = 36.

11. Subtract the first number from the second number to get the

third number in each row.

First Row 30 − 9 = 21

Second Row ?−6 = 14 ⇒= 14 + 6 = 20

Third Row 40 − 12 = 28.

12. The sum of the upper right number and the lower left

number is equal to the lower right number while their

product gives the upper left number.

First arrangement 7 + 4 = 11 and 7 × 4 = 28

Second arrangement 5 + 5 = 10 and 5 × 5 = 25

Third arrangement 8 + 3 = 11 and 8 × 3 = 24.

13.

AF = AB + BF = 24 + 15 ft = 39 ft.

14. Husband ⇒ One , Wife ⇒ One , Five married sons ⇒ 5 ×2=10

Number of children ⇒5×4=20

total number of members = 1+1+10+20=32.

15. India is famous for Mango.

Similarly, Kiwis are found in New Zealand.

16. V→E, Z→A, S→H Pairs of Opposite Letters

Similarly, Q→J, U→F, N→M

17. 42 − 22 = 20; Similary, 64 − 22 = 42.

18. Except apple, all other are juicy fruits.

19. Except the number 632, all other numbers are completely

divisible by 5.

725

5= 145;

840

5= 168;

475

5= 95; But

632

5= 126.4.

20. In the number pair ’64-36’, both the numbers are perfect

squares. 64 = 8 × 8; 36 = 6 × 6.

21. Meaningful order of words: (3) Open→(1) Type→(4)

Save→(2)Print→(5)Close.

22.

The rectangles are: IJPO; JKQP; IKNL; LNQO; ABFG; BCEF;

ACDH; HDEG Squares are also rectangles; ABMH; BCDM;

HMFG; MDEF; IJML; JKNM; MNQP; LMPO; ACEG; IKQO.

23. Some lecturers may be doctors and vice-versa. Some

Government servants may be lecturers and vice-versa. Some

Government servants may be doctors and vice-versa. Some

Government servants who are lecturers may be doctors.

some doctors who are lecturers may be Government

servants.

24.

25. Do yourself

26. Here, enquired about/appealed for… should be used. Enquire

= to ask somebody for information.

27. Have a part to play = to be able to help.

Hence, each have a part to play … should be used.

28. Here, a ten rupee (singular ) note… should be used.

Numeral Adjective+Hyphen+singular Noun two five-rupee

notes.

29. Here, heard of the earthquake… should be used.

31. Around the clock = all day and all night without stopping.

32. Come to light = to become known to people.

Look at the sentence: New evidence has recently come to

light.

37. Jealous (Adjective) = feeling anger or unhappy because you

wish you had something that somebody else has; envious.

Look at the sentence: She is jealous of my success.

TARGET NO-1 CLASSES (A UNIT OF) EXECUTIVE MAKERS PVT.LTD. V-157

FOR DOWNLOADING BOOKS : www.executivemakers.com Page-30

38. Futility (Noun) = uselessness; having no purpose;

pointlessness.

39. Prudent (Adjective) = sensible and careful when you make

judgment and decisions; avoiding unnecessary risks.

40. Turbulent (Adjective) = unruly; changing direction suddenly

and violently; a situation in which there is lot of sudden

change, confusion, disagreement etc.

Calm (Adjective) = not upset; not excited or nervous.

51. Marks obtained by A in English = 2x (let)

Marks obtained in Maths = 3x

Marks obtained in Science = x

2𝑥 + 3𝑥 + 𝑥 = 180 ⇒ 6𝑥 = 180 ⇒ 𝑥 =180

6= 30

= Mark obtained in science.

52. Total age of 5-member family 3 years ago = 17×5=85 years.

Their total present age = 85+3×5=100 years

Total present age of 6 members = 17×6=102 years

Present age of child = 102-100=2 years.

53. C.P. of 50 pairs of shoes = Rs. (50×189.5) =Rs. 9475.

Their S.P. = Rs. 10000 ,Gain = Rs. (10000-9475) = Rs. 525.

54. 𝑥% 𝑜𝑓 𝑦 =xy

100, y −

xy

100= y 1 −

x

100 .

55. Population of the illiterate in the village =

100 − 30 % 𝑜𝑓 6600 = 6600 × 70

100= 4620.

56. Let both trains meet after t hours.

Distance = Speed × Time = 60𝑡 − 50𝑡 = 120 ⇒ 10𝑡 =

120, 𝑡 = 12 hours. Required distance = 60t + 50t = 110t =

110 × 12 = 1320 km.

57. S.I. = Principal×Time×Rate

100=

32000×4×10

100= Rs. 12800

C.I. = P [(1 +R

100)4 − 1] = 32000 1 +

10

100

4

− 1 =

32000 1.1 4 − 1 = 32000 1.4641 − 1 = Rs. 14851.2

Required Difference = 14851.2-12800= Rs. 2051.2.

58. percentage increase in area = (𝑥 + 𝑦+𝑥𝑦

100)%,

Here, x = 100%; y = 100%

100 + 100 +100×100

100 % = 300%.

59. Volume of metallic sphere = 4

3πr3 =

4

3× π × 3 × 3 × 3 =

36π cu. cm. Volume of cone = 36π cu. cm.⇒1

3πR2h = 36π

⇒ R2h = 108 ⇒ 6 × 6 × h = 108 ⇒ h =108

6 × 6= 3 cm.

60. 𝑎 + 𝑏 + 𝑐 = 0 Given a + b = −c, b + c = −a, c + a =

−b , (𝑎 + 𝑏 − 𝑐)2 + (𝑏 + 𝑐 − 𝑎)2 + (c + a − b)2 =

(−c − c)2 + (−a − a)2 + (−b − b)2 = (−2c)2 + (−2a)2 +

(−2b)2 = 4c2 + 4a2 + 4b2 = 4 c2 + a2 + b2 .

61. 𝑥 − 𝑦 = 2015 − 2014 = 1, 𝑦− 𝑧 = 2014 − 2013 = 1, 𝑧 − 𝑥 =

2013 − 2015 = −2, 𝑥2 + 𝑦2 + 𝑧2 − 𝑥𝑦 − 𝑦𝑧 − 𝑧𝑥 =1

2 2𝑥2 + 2𝑦2 + 2𝑧2 − 2𝑥𝑦 − 2𝑦𝑧 − 2𝑧𝑥 =

1

2 𝑥2 + 𝑦2 − 2𝑥𝑦 +

�2+�2−2��+�2+�2−2�𝑥=12(�−�)2+(�−�)2+(�−�)2=121+1+4=1

2×6=3.

62. 𝑥 +1

𝑥= 2

1

12=

25

12 On squaring both sides x +

1

x

2

= 25

12

2

x2 +1

x2 + 2 =625

144⇒ x2 +

1

x2 =625

144− 2 ⇒

625−288

144=

337

144⇒

x −1

x

2

+ 2 =337

144⇒ x −

1

x

2

=337

144− 2 =

337−288

144=

49

144. ⇒

𝑥−1

𝑥=

49

144=

7

12 ,𝑥4 −

1

𝑥4 = 𝑥2 +1

𝑥2 𝑥2 −

1

𝑥2 =

𝑥2 +1

𝑥2 𝑥 +1

𝑥 𝑥 −

1

𝑥 =

337

144×

25

12×

7

12=

58975

20736.

63.

Point ‘O’ is centroid and AD is median. AO=2

3AD ⇒ 10 =

2

3AD ⇒ AD =

10×3

2= 15cm, OD =

1

3AD =

15

3= 5cm.

64.

BDC = 120° ,CDP = 180°-120°=60°

CD II AB ABP=60° = CDP.

65.

Required length of tangent =

(distance between centres)2 − (r1 − r2)2 =

(25)2 − (16 − 9)2 = 625 − 49 = 576 = 24 cm.

66. sin θ + 18° = cos 60° = cos 90° − 30° = sin 30° ⇒ θ +

18° = 30° ⇒ θ = 30° − 18° = 12° , cos 5θ = cos 60° =1

2.

67. tan θ =3

4⇒ tan2 θ =

9

16 Expression =

4sin2 θ−2cos2θ

4sin2θ+3cos2θ=

4sin2θ

cos2θ−2

cos2θ

cos2θ

4sin2θ

cos2θ+3

cos2θ

cos2θ

=4tan2θ−2

4tan2θ+3=

4×9

16−2

4×9

16+3

=9

4−2

9

4+3

=9−8

9+12=

1

21.

68.

ABCD is a concyclic quadrilateral. A + C = B + D =

180° ,A = 180° − C, cos A = cos(180° − C) =

TARGET NO-1 CLASSES (A UNIT OF) EXECUTIVE MAKERS PVT.LTD. V-157

FOR DOWNLOADING BOOKS : www.executivemakers.com Page-31

− cos C and cos B = − cos D , cos A + cos B − cos A −

cos B = 0.

69. Product of two numbers = HCF × LCM ⇒ Numbers = zx and zy.

zx × zy = z × LCM ⇒ LCM = xyz.

70. 12 men = 24 boys ,1 man = 2 boys ,15men + 6 boys = 36 boys

M1D1 = M2D2 ⇒ 24 × 66 = 36 × D2 ⇒ D2 =24×66

36=

44 𝑑𝑎𝑦𝑠 .

71.

AC= 24 cm = d2 , BD = 32 cm = d1 , OD = 16 cm, OC =

12 cm, COD = 90° , CD = OC2 + OD2 = 122 + 162 =

144 + 256 = 400 = 20 cm

Perimeter of rhombus = 4× CD = 4×20=80 cm.

72. Number of examinees getting more than average marks =

72 + 48 + 24 + 8 = 152 .

73. Number of students who got above 80% marks =24 + 8 =

32. Required percent = 32

273× 100 = 11.72%.

74. Number of students who got marks above 60% and below

80% = 72 + 48 = 120 ,Required percent = 120×100

273= 43.95%.

75. Number of students who got 40% or less marks = 2 + 4 +

12 + 26 = 44 ,Required Percentage = 44

273× 100 = 16.11%.

76. The Sahara is the largest subtropical hot desert and third

largest desert after Antarctica and the Arctic. At over 9,

400,000 Square kilometers, it covers most of North Africa,

making it almost as large as China or the United States. In

terms of area, the Arabian and the Kalahari deserts come at

second and third places among subtropical deserts.

77. Above Troposphere, Temperature increases rapidly with

Height in the Stratosphere layer as it houses the ozone layer

which absorbs ultraviolet (UV) rays from the sun. Again,

temperature increases rapidly with height in Thermosphere

which comprises Ionosphere (lower part) and Exosphere

(upper part). Temperature in Exosphere which is the

uppermost layer of the atmosphere reaches up to more than

4500° Celsius.

78. There are three basic classes of photosynthetic pigments:

chlorophylls (greenish pigments), Carotenoids (usually red,

orange, or yellow pigments) and Phycobilins (water-soluble

pigments found in the cytoplasm) .Anthocyanins play no part

in Photosynthesis.

79. Keratin is a family of fibrous structural proteins. It is the key

structural material making up the outer layer of human skin.

It is also the key structural component of hair and nails.

Another example of fibrous protein is collagen.

80. Swedish naturalist Carl Linnaeus established the binomial

system of naming living organisms, setting a format and a

structured process for classifying the interrelationships

between plants (among which he included all fungi), and

between animals. The binomial system that Linnaeus devised

enables an author to refer to a species confident that it will

mean the same thing to inform readers anywhere else in the

world.

81. System management is designed to balance processing and

input/output, with the aim of providing reasonable minimum

turnaround and maximum throughput. Turn around is the

amount of time elapsing between a request to the computer

to execute a program or input and the availlability of output.

82. GARUDA (Global Access to Resource Using distributed

Architecture) is the other name of the Indian National Grid

Computing Initiative. It is a collaboration of scientific and

technological researchers on a nationwide grid comprising of

computational nodes, mass storage and scientific

instruments.

83. The maximum density of water occurs at 4°C because at this

temperature two opposing effects are in balance. Water is an

exceptional compound that has a lower density in the solid

phase than the liquid phase (i.e., ice floats on water). This is

actually due to the ion nature of water in the liquid phase,

which packs the molecules tightly together.

84. Human Rights Day is celebrated annually across the world on

10 December. The day commemorates the United Nation

General Assembly’s adoption and proclamation, on 10

December 1948, of the Universal Declaration of Human

Rights.

85. The Ajanta Caves in Aurangabad district of Maharashtra ,are

about 30 rock-cut Buddhist cave monuments which date from

the 2nd

century BC to about 480 or 650 A.D. Most of them

were constructed during Gupta Period.

86. Bharat Mata is an epic painting by celebrated Indian painter,

Abanindranath Tagore. This painting depicts Bharat Mata as

Lakshmi, the Goddess of Plenty, clad in the apparel of a

Vaishnava nun.

87. Rostow’s ‘Stages of Economic Growth’ (1960) presented five

stages through which all countries must pass to become

developed: (1) traditional society, (2) preconditions to take-

off, (3) take-off, (4) drive to maturity, and (5) age of high mass

consumption. Take-off is the short period of intensive

growth, in which industrialization begins to occur, and

workers and institutions become concentrated around a new

industry.

88. July-December corresponds to the main agricultural season in

India, while January-June is a relatively ‘slack’ season. Since

more than 50% of India’s workforce is engaged in agriculture

sector, the slackness of this sector imparts sluggishness to the

entire economy.

89. The 73rd

Amendment of the Constitution was enacted in

1992, to enshrine in the Constitution certain basic and

TARGET NO-1 CLASSES (A UNIT OF) EXECUTIVE MAKERS PVT.LTD. V-157

FOR DOWNLOADING BOOKS : www.executivemakers.com Page-32

essential features of Panchayat Raj Institutions. It introduced

a three-tier Panchayati Raj system at the Village,

Intermediate (Block) and District levels.

90. The Panchayat Raj system was first adopted by Nagaur

district of Rajasthan on 2 October, 1958,. Andhra Pradesh

soon followed the suite. Panchayti Raj is the oldest system of

local government in the Indian subcontinent.

91. The capital of Mughal empire was changed from Agra to Delhi

(traditionally known as Shahjahanabad ) by Shahjahan in the

year 1638. The tenure of Shahjahan was 1628-1658 A.D.

92. Woodrow Wilson quipped,”A living thing is born” after the

League Covenant was drafted in 1919. The League was an

intergovernmental organization founded on 10 January,

Conference that ended the First World War.

93. The ‘Arthashastra’was authored by Kautilya, also known as

Vishnugupta, who was a contemporary and mentor of

Chandragupta Maurya, founder of the Mauryan Empire. It is

an ancient Indian treatise on statecraft, economic policy and

military strategy, written in Sanskrit.

94. The Taj Mahal was built by Mughal emperor Shahjahan in

memory of his third wife, Mumtaz Mahal in 1632-1653. It is a

white marble mausoleum located in Agra, Uttar Pradesh. It is

a UNESCO World Heritage Site and widely recognized as “The

jewel of Muslim art in India.”

95. As in most other contemporary civilizations, agriculture was

the backbone of the Indus Economy. The people made

extensive use of the wooden plows. Barley and wheat were

the main food crops. Agriculture sustained the rudimentary

urban centers that emerged (Studies in World History Volume

1 by James P. Stobaugh).

96. Bartholomew Dias, a Portuguese voyager, discovered the

Cape of Good Hope in Africa in December 1487. He had

named the Cape “the Cape of Storms”(Cabo das Tormentas),

But King John II of Portugal later renamed it Cabo de Boa

Esperanca (Cape of Good Hope).

97. Sodium carbonate (Na2CO3) is known as washing soda, soda

ash and soda crystals. It is a salt of carbonic acid, a chemical

that produces a wide range of salts collectively known as

carbonates. It can be used to remove stubborn stains from

laundry.

98. Under Article-324 of the Indian Constitution, the

superintendence, direction and control of elections to both

Houses of Parliament and State Legislatures is vested in the

Election Commission if India. The same article authorizes the

Commission to conduct the elections to the offices of the

President and Vice-President of India.

99. Typhoid fever is an infection that causes diarrhea and a rash.

It is most commonly due to a type of bacterium called

Salmonella typhi (S. typhi). S.typhi is spread through

contaminated food, drink or water.

100. Africa is located in the Northern, southern, Eastern, and

Western Hemispheres. The equator (divides the earth into

the Northern and Southern Hemispheres) and the Prime

Meridian (divides the earth into the Eastern and Western

Hemispheres) pass through Africa.

PRACTICE SET-5 General Intelligence & Reasoning

Directions (1-2) : In each of the following questions, select the related word/letters from the given alternatives. 1. Blue whale : Sea : : ? : Land

(1) Turtle (2) Fish (3) Elephant (4) Green whale

2. Hard : Rock : : Soft : ? (1) Lead (2) Paper (3) Sponge (4) Glass

Directions (3-4) : In each of the following questions, find the odd word/number/letters from the given alternatives. 3. (1) Sparrow (2) Kingfisher

(3) Nightingale (4) Bat 4. (1) Shack (2) Barn

(3) Stable (4) Barrack Directions (5-6) : In each of the following questions, a series is given, with one term missing. Choose the correct alternative from the given ones that will complete the series. 5. STU, WXY, ABC, ?

(1) DEF (2) EFG (3) FCG (4) EGF

6. GX, JU, NR, SO, ? (1) VL (2) YL (3) XB (4) YA

7. Find the wrong term in the following series : 49, 49, 50, 54, 60, 70, 104 (1) 60 (2) 49 (3) 104 (4) 54

8. Among the four sets, find out the set, which is like the given set. Given set: (12, 72, 216) (1) (4, 24, 48) (2) (7, 42, 252) (3) (12, 60, 30) (4) (5, 30, 90)

9. A series of figures are given which can be grouped as related to students, artists and scientists. Select the groups into which the figures can be classified. (1) Pen (2) Flask (3) Book (4) Test tube (5) School bag (6) Design (7) Flame (8) Flower (9) Painting

(1) (1, 3, 5) (2, 4, 7) (6, 8, 9) (2) (2, 3, 5) (1, 6, 7) (4, 8, 9) (3) (1, 2, 6) (3, 4, 8) (7, 5, 9)

TARGET NO-1 CLASSES (A UNIT OF) EXECUTIVE MAKERS PVT.LTD. V-157

FOR DOWNLOADING BOOKS : www.executivemakers.com Page-33

(4) (3, 4, 5) (1, 2, 6) (7, 8, 9) 10. Which one of the following numbers lacks the common

property in the series? 81, 36, 25, 9, 5, 16

(1) 5 (2) 9 (3) 36 (4) 25

11. A car travels 20 miles in the same time as another car, travelling 20 MPH faster, covers 30 miles. How long does the journey take?

(1) 31 minutes (2) 29 minutes (3) 30 minutes (4) 28 minutes

12. A person walks 9 km to the South. From there he walks 5 km to the North. After this he walks 3 km to the West. In which direction and how far is he now from the starting point?

(1) 4 km South (2) 4 km North (3) 5 km North-West (4) 5 km South-West

13. Shankar and Ganesh walked 150 metres towards East. Shankar turned left and walked 100 metres. Ganesh went straight for another 100 metres. Afterwards Ganesh turned to right and Shankar turned left. In which direction are they facing now?

(1) East and North (2) West and South (3) North and South (4) South and North

14. Complete the third equation on the basis of a certain system followed in the first two equations.

1. 1 8 5 3 7 = 73581

2. 5 7 6 2 4 = 42675

3. 9 4 3 2 8 = ? (1) 83924 (2) 82349 (3) 28394 (4) 28934

15. Identify the hidden name of the geometrical figure in the given word from among the answer words whose letters are given in jumbled form. SPIRITUAL.

(1) RISPAL (2) RITUAL (3) SPIRIT (4) RIPITU

Directions (16-17) : In each of the following questions, select the missing number from the given responses. 16. W

8 7 6

8 7 6

88 77 ?

5632 3773 3132

(1) 66 (2) 87 (3) 78 (4) 76

17.

28 35 32

7 5 8

4

8

3

10

?

9

(1) 3 (2) 5 (3) 6 (4) 7

18. In the alphabetical order A, B, C, D, .....Z, which letter is 11

th to the left of the letter which is 4

th to the right of T?

(1) K (2) O (3) I (4) M

19. If DEGI is equal to 25, what is FEHD equal to ? (1) 32 (2) 25 (3) 52 (4) 23

20. If SISTER is coded as 20, 10, 20, 21, 6, 19, then the code for BROTHER is

(1) 2, 15, 16, 21, 9, 5, 18 (2) 3, 19, 16, 21, 9, 6, 19 (3) 4, 20, 15, 18, 8, 7, 9 (4) 3, 18, 16, 20, 9, 7, 19

21. If 64 + 53 = 4, 86 + 42 = 4, then 83 + 72 = ? (1) 12 (2) 10 (3) 15 (4) 18

22. How many triangles are there in the figure ABCDEF?

(1) 24 (2) 26 (3) 28 (4) 30

23. A square sheet of paper has been folded and punched as shown below in the question figures. You have to figure out from amongst the four answer figures, how it will appear when opened? Question Figures :

Answer Figures :

(1) (2)

(3) (4) 24. If America is called Greenland, Greenland is called Africa,

Africa is called Russia, Russia is called India and India is called Pakistan; Delhi is called the capital of which country?

(1) Russia (2) India (3) Pakistan (4) Greenland

25. Which number space indicates Indian teachers who are also advocates?

(1) 2 (2) 3 (3) 4 (4) 6

TARGET NO-1 CLASSES (A UNIT OF) EXECUTIVE MAKERS PVT.LTD. V-157

FOR DOWNLOADING BOOKS : www.executivemakers.com Page-34

General Awareness

26. ‘Economic Survey of India’ is published every year by (1) Ministry of Commerce (2) C.S.O (3) Ministry of Finance (4) Ministry of Economic Affairs

27. The term ‘food security’ implies (1) sufficient supplies of food at affordable prices (2) abundance of food with assurance to enjoy (3) availability of food of superior and nutrition quality (4) balanced food with secure conditions

28. Which one of the following factors does not shift the demand curve for a product to the right?

(1) Successful advertising (2) A fall in the price of its complements (3) A rise in the price of its substitutes (4) A fall in the price of the product itself

29. The Indian Constitution envisages not only a democratic form of government but also a democratic society, because it is infused with the spirit of

(1) Justice (2) Liberty (3) Equality (4) fraternity Which of the correct answer code? (1) 1 and 2 (2) 2 and 3 (3) 1 and 3 (4) 1, 2, 3 and 4

30. Which one of the following States enjoys the distinction of being the first linguistic State of India?

(1) West Bengal (2) Andhra Pradesh (3) Tamil Nadu (4) Punjab

31. What was the other name of Nana Sahib? (1) Ramachandra Panduranga (2) Dhondu Pant (3) Tantia Tope (4) Kunwar Singh

32. Who was the Russian leader when the Indo-Soviet Treaty of Friendship and Co-operation was signed between India and Russia?

(1) Nikita Khrushchev (2) Leonid Brezhnev (3) Yuri Andropov (4) Mikhail Gorbachev

33. The greatest Portuguese Governor who laid the real foundation of Portuguese power in India was

(1) Almeida (2) Albuquerque (3) Francis Drake (4) Vasco-da-Gama

34. Which of the following is the leading sediment transporting river in India?

(1) Gangas (2) Indus (3) Brahmaputra (4) Yamuna

35. Which of the following statements is correct? (1) Capacity of air to hold moisture decreases with a rise in temperature of air. (2) The air pressure decreases with an increase in

temperature . (3) Absolute humidity is expressed in grams per cubic metre of air. (4) With an ascent of every 165 metres, the temperature is reduced by 1

0C.

36. A plant bearing both male and female flowers is called (1) bisexual (2) dioecious

(3) monoecious (4) monogamous 37. ‘Jhum’ is

(1) a folk dance (2) the name of a river (3) a tribe in the North-East of India (4) a type of cultivation

38. Quinine is obtained from a plant. What part of the plant yields the drug?

(1) Leaves (2) Fruits (3) Root (4) Stem bark

39. The computer memory holds data (1) bytes (2) program (3) registers (4) bits

40. Math the following

Alloy Composition

A. Bronze 1. Pb + Sb + Sn

B. Brass 2. Ni + Zn + Cu

C. German Silver 3. Cu + Zn

D. Type metal 4. Cu + Sn

A B C D (1) 1 4 3 2 (2) 2 1 4 3 (3) 3 2 1 4 (4) 4 3 2 1

41. It is not advisable to sleep under a tree at night because of the

(1) release of less oxygen. (2) release of more oxygen. (3) release of carbon dioxide. (4) release of carbon monoxide.

42. The calorie requirement of the body increases in winter as compared to summer because more calories are necessary to

(1) make more fat in the body. (2) compensate for falling hair. (3) sustain body temperature. (4) break more proteins.

43. Match the following A. Copper sulphate B. Penicillin C. Urea D. Malathion

(1) Fertiliser (2) Insecticide (3) Fungicide (4) Antibiotic

A B C D (1) 1 2 3 4 (2) 2 4 1 3 (3) 3 4 1 2 (4) 4 3 2 1

44. When plant diversity is maintained in natural habitat, the conservation is called

(1) in vivo (2) in vitro (3) ex situ (4) in situ

45. What does the term ‘Lithosphere’ refer to ? (1) Plants and animals (2) Interior of the earth (3) Crust of the earth (4) None of the above

46. Which part of throat in men is called Adam’s apple?

TARGET NO-1 CLASSES (A UNIT OF) EXECUTIVE MAKERS PVT.LTD. V-157

FOR DOWNLOADING BOOKS : www.executivemakers.com Page-35

(1) Cricoid cartilage (2) Larynx (3) Bronchi (4) Thyroid cartilage

47. GSLV Project of India is connected with (1) agriculture development. (2) conservation of river water. (3) banking system. (4) missile engines for space programmes.

48. When was Olympic Flag designed ? (1) 1913 (2) 1914 (3) 1915 (4) 1916

49. Which country has hosted the 2012 Olympic Games? (1) U.K. (2) Germany (3) U.S.A. (4) South Korea

50. When was first Gyanpeeth award given in literature? (1) 1965 (2) 1966 (3) 1967 (4) 1968

Quantitative Aptitude

51. The number of times 2 is used while writing the number 1 to 100 is

(1) 21 (2) 20 (3) 19 (4) 18

52. If the product of two numbers is 2160 and HCF is 6, then the ratio of HCF and LCM is

(1) 21 : 60 (2) 60 : 21 (3) 1 : 60 (4) 60 : 1

53. The value of 0.2304 + 0.1764

0.2304− 0.1764 is

(1) 6 (2) 28 (3) 18 (4) 15

54. A tank can be filled by pipe A in 2 hours and by pipe B in 3 hours. An outlet C can empty the tank in 6 hours. If all the three are opened simultaneously, the tank will be filled in

(1) 1 hours (2) 11

2 hours

(3) 2 hours (4) 11

3 hours

55. If a shopkeeper marks the price of goods 50% more than their cost price and allows a discount of 40%, what is his gain or loss per cent?

(1) Gain 10% (2) Gain 15% (3) Loss 15% (4) Loss 10%

56. The weights of zinc, copper and aluminium in an alloy are in the ratio 2 : 3 : 7. In the alloy weighing 48 kg, the difference in weight of zinc and aluminium is

(1) 5 kg (2) 16 kg (3) 20 kg (4) 4 kg

57. The average weight of 8 persons is increased by 2.5 kg when one of them whose weight is 56 kg is replaced by a new man. The weight of the new man is

(1) 20 kg (2) 64 kg (3) 58.5 kg (4) 76 kg

58. The difference between the selling price of a shirt sold at profits 15% and 17% is Rs. 3. Then the cost price if the shirt is

(1) Rs. 200 (2) Rs. 150 (3) Rs. 175 (4) Rs. 180

59. Ratan’s expenditure and savings are in the ratio 5 : 1. If his salary is increased by 25% and savings by 15%, the percentage increase in his expenditure is

(1) 18% (2) 27% (3) 28% (4) 32%

60. Cyclist A started his journey on cycle at 7.30 a.m. at a speed of 8 km/hr. After 30 minutes, cyclist B started from the same place but with a speed of 10 km/hr. At what time did B overtake A?

(1) 8 a.m. (2) 10 a.m. (3) 9 a.m. (4) 9.30 a.m.

61. If the simple interest on a certain sum of money for three years is Rs. 225 and the compound interest on the same sum at the same rate for 2 years is Rs. 153, then the principal invested, in rupees, is

(1) 1,500 (2) 2,250 (3) 3,000 (4) 1,875

62. The areas of a square and a rectangle with equal perimeter are denoted by S and R respectively. Which one of the following is correct?

(1) S = R (2) S > R (3) S < R (4) S = 2R

63. Semi-vertical angle of a right circular cone is 300. If the

ratio of the numerical value of the volume and slant surface area be 1 : 3, the radius of the base is

(1) 1 units (2) 2

3 𝑢𝑛𝑖𝑡𝑠

(3) 3 units (4) 3 units 64. Each of the five angles of a polygon is equal to 172

0 and

the other angles are 1600 each. The number of sides of

the polygon is (1) 20 (2) 21 (3) 22 (4) 23

65. The volume of a solid hemisphere is numerically equal to its total surface area. Its radius is

(1) 3 units (2) 11

2 units

(3) 4 1

2 units (4) 9 units

66. If x = 2t and y = 2𝑡−1

3, then what is the value of t for which

x = y holds?

(1) 1

4 (2) −

1

2

(3) 1

3 (4) −

1

4

67. If x = 5+1

5−1, value of x

2 + x – 1, is

(1) 5 (2) 2

(3) - 5 (4) 5 + 1 68. Equations of the straight lines containing two sides of a

right-angled triangle are given by y = x and y = 0. of the following, the equation which cannot represent the third side is

(1) x = 5 (2) x = 2 (3) x = 0 (4) x = -3

69. In the figure below, lines k and l are parallel. The value of a

0 + b

0 is

TARGET NO-1 CLASSES (A UNIT OF) EXECUTIVE MAKERS PVT.LTD. V-157

FOR DOWNLOADING BOOKS : www.executivemakers.com Page-36

(1) 45

0 (2) 100

0

(3) 1800 (4) 360

0

70. SR is a direct common tangent to the circles of radii 8 cm and 3 cm respectively, their centres being 13 cm apart. If the points S and R are the respective points of contact, then the length of SR is

(1) 12 cm (2) 11 cm (3) 17 cm (4) 10 cm

71. The bisector of the angle BAC of a triangle ABC intersects the side BC at the point D and meets the circumcircle of

the ABC at E. Them, it is always true that AB.AC + DE.AE = ?

(1) AD2 (2) AE

2

(3) CE2 (4) CD

2

Directions (72-75) : The following line-graph gives the annual per cent profit earned by company during the period 1995-2000. Study the line-graph and answer the questions.

% Profit = 𝐼𝑛𝑐𝑜𝑚𝑒 −𝐸𝑥𝑝𝑒𝑛𝑑𝑖𝑡𝑢 𝑟𝑒

𝐸𝑥𝑝𝑒𝑛𝑑𝑖𝑡𝑢𝑟𝑒

72. Profit earned by the company is maximum in the year

(1) 1995 (2) 1999 (3) 2000 (4) 1997

73. The period in which the profit of the company has increased fastest is

(1) 1997-98 (2) 1995-96 (3) 1998-99 (4) 1996-97

74. The range of profit made by the company is (in percentile)

(1) 25 (2) 20 (3) 15 (4) 30

75. The expenditure of the company during the year 1995 was Rs. 20 thousand crore. Then the income of the company in that year was (in thousand crore Rs.)

(1) 15 (2) 28 (3) 25 (4) 12

English Language

Directions (76-77) : In the following questions, some part of the sentences have errors and some have none. Find out which part of a sentence has an error. The number of that part is your answer. If there is no error, your answer is (4) i.e., No error.

76. Having lived (1) / in Kerala for ten years, (2) / my friend is used to speak Malayalam with his friends. (3) / No error (4)

77. Much water (1) / has flown (2) / under this bridge. (3) / No error (4)

Directions (78-80) : In the following questions, sentences are given with blanks to be filled in with an appropriate word (s). Four alternatives are suggested for each question. Choose the correct alternative out of the four. 78. He will dispense ... your services.

(1) of (2) off (3) on (4) with

79. I have given her ........ (1) a work (2) some works (3) a piece of work (4) many works

80. He is a popular teacher. He seems to be ..... for that profession.

(1) cut down (2) cut off (3) cut in (4) cut out

Directions (81-83) : In the following questions, out of the four alternatives, choose the one which best expresses the meaning of the given word. 81. Indiscriminate

(1) undifferentiated (2) instant (3) sensible (4) discreet

82. Literal (1) verbatim (2) formal (3) idealistic (4) outdated

83. Intricate (1) puzzling (2) illusive (3) complicated (4) assertive

Directions (84-87) : In the following questions, choose the word opposite in meaning to the given word. 84. Resourcefulness

(1) scarcity (2) stupidity (3) incompetence (4) bankruptcy

85. Evolve (1) withdraw (2) withhold (3) suspend (4) stop

86. Antiquated (1) renewed (2) unique (3) modern (4) renovated

87. Pathetic (1) comic (2) ridiculous (3) dramatic (4) trivial

Directions (88-90) : In the following questions, four alternatives are given for the Idiom/Phrase printed in bold in the sentence. Choose the alternative which best expresses the meaning of the Idiom/Phrase. 88. For some people, writing verse is as duck takes to water.

(1) like taking the duck to water. (2) like bursting out suddenly. (3) like dropping the duck in the water. (4) like easily and naturally speaking.

89. He made my day by telling me how important I was to him.

(1) gave me great pleasure (2) displeased me (3) spoiled my day

TARGET NO-1 CLASSES (A UNIT OF) EXECUTIVE MAKERS PVT.LTD. V-157

FOR DOWNLOADING BOOKS : www.executivemakers.com Page-37

(4) made me resentful 90. He made away with ten thousand rupees in the course

of three months. (1) earned (2) ran away with (3) squandered (4) saved

Directions (91-95) : In the following questions, a part of the sentence is printed in bold. Below are given alternatives to the bold part at (1), (2) and (3) which may improve the sentence. Choose the correct alternative. In case no improvement is needed, your answer is (4) 91. The ship ran over when it crashed into an iceberg.

(1) got over (2) gave up (3) went down (4) No improvement

92. His wife was contentious. (1) contagious (2) quarrelsome (3) content (4) No improvement

93. Historians feel there is an earnest need for the review of history text books every five years and a revision of the same every ten years.

(1) imperative (2) indispensable (3) urgent (4) No improvement

94. My car broke off on my way to the office. (1) out (2) in (3) down (4) No improvement

95. Freedom is a wonderful thing, for Jimmy was eager to experience it.

(1) though (2) and (3) but (4) No improvement

Directions (96-100) : In the following questions, out of the four alternatives, choose the one which can be substituted for the given words/sentence. 96. An act or notion to look back in the past.

(1) Retrospective (2) Postnatal (3) Retrogressive (4) Primitive

97. Medicine to counteract the effect of a poison. (1) Emetic (2) Antidote (3) Anti-venom (4) Antiseptic

98. A collection of poems. (1) Pathology (2) Anthology (3) Oncology (4) Pedology

99. One who studies mankind. (1) Anthropologist (2) Physicist (3) Pathologist (4) Philanthropist

100. An opinion contrary to popular belief. (1) Paradox (2) Orthodoxy (3) Hearsay (4) Heresy

TARGET NO-1 CLASSES (A UNIT OF) EXECUTIVE MAKERS PVT.LTD. V-157

FOR DOWNLOADING BOOKS : www.executivemakers.com Page-38

ANSWER SHEET

1.3 2.3 3.4 4.1 5.2 6.2 7.1

8.4 9.1 10.1 11.3 12.4 13.2 14.2

15.1 16.2 17.2 18.4 19.4 20.2 21.2

22.3 23.2 24.3 25.2 26.3 27.1 28.2

29.4 30.2 31.2 32.1 33.2 34.1 35.4

36.3 37.4 38.4 39.2 40.4 41.3 42.3

43.3 44.4 45.3 46.4 47.4 48.1 49.1

50.1 51.2 52.3 53.4 54.2 55.4 56.3

57.4 58.2 59.2 60.2 61.4 62.2 63.2

64.4 65.3 66.4 67.4 68.4 69.1 70.1

71.2 72.2 73.1 74.4 75.2 76.3 77.2

78.4 79.3 80.4 81.1 82.1 83.3 84.2

85.2 86.3 87.3 88.4 89.1 90.2 91.3

92.2 93.4 94.3 95.2 96.1 97.2 98.2

99.1 100.4

TARGET NO-1 CLASSES (A UNIT OF) EXECUTIVE MAKERS PVT.LTD. V-157

FOR DOWNLOADING BOOKS : www.executivemakers.com Page-39

Practice Set – 5

Hints & Solutions

1. The abode of Blue Whale is sea.

Similarly, the abode of Elephant is land.

2. The first term is the property of the second term. Rock

remains hard. Similarly, sponge is soft.

3. Except Bat, all other are birds: Bat is a mammal.

4. Shack is roughly built shed or hut.

Barn is “a large simple building for storing grain,: “a building

for sheltering farm animals” etc.

Stable is a building in which horses are kept and fed.

Barrack refers to a large building or group of buildings for

soldiers to live in.

5. S(+1)→T(+1)→U, W(+1)→X(+1)→Y, A(+1)→B(+1)→C,

E(+1)→F(+1)→G.

6. G(+3)→J(+4)→N(+5)→S(+6)→Y

X(-3)→U(-3)→R(-3)→O(-3)→L.

7. 49 + 0 = 49, 49 + 1 = 50, 50 + 4 = 54, 54 + 9 = 63, 63 +

16 = 79, 79 + 25 = 104.

Thus, the number 60 is wrong in the series.

8. 12 × 6 = 72; 72 × 3 = 216;

5 × 6 = 30; 30 × 3 = 90.

9. Students ⇒ Figures 1, 3 and 5 , Artists ⇒ Figures 6, 8 and 9

Scientists ⇒ Figures 2, 4 and 7.

10. Except the number 5, all other numbers are prefect squares.

81 = 9 × 9; 36 = 6 × 6,25 = 5 × 5; 9 = 3 × 3; 16 = 4 × 4.

11. Difference in the distances covered by both the cars =

30 − 20 miles = 10 miles

the faster car travelling 20 MPH faster, covers 10 miles more .

Therefore, the time of journey = 10

12=

1

2hour.

12.

Required Distance = AD = (AC)2 + (CD)2 = (4)2 + (3)2 =

16 + 9 = 25 = 5 km.

13.

Shankar is facing towards West. Ganesh is facing towards

Sough.

14. The digits have been written in reverse order. 1 1 × 8 ×

5 × 3 × 7 = 73581, 2 . 5 × 7 × 6 × 2 × 4 = 42675 3 . 9 ×

4 × 3 × 2 × 8 = 82349.

15. RISPAL ⇒SPIRAL.

16. First column 8 × 8 × 88 = 5632, Second column 7 × 7 × 77 =

3773, Third column 6 × 6 ×? = 3132 , ? =3132

36= 87.

17. First column 28

7= 4; 4 + 4 = 8 Second column

35

5= 7; 7 + 3 =

10 , Third column32

8= 4; 4 + 5 = 9.

18. 4th

to the right of T⇒X , 11th

to the left of X⇒M.

19. 4+5+7+9 = 25, 6+5+8+4 = 23

20. S ⇒ 19 + 1 = 20; I ⇒ 9 + 1 = 10; S ⇒ 19 + 1 = 20; T ⇒ 20 +

1 = 21; E ⇒ 5 + 1 = 6; R ⇒ 18 + 1 = 19.

Similarly,

B ⇒ 2 + 1 = 3; R ⇒ 18 + 1 = 19; O ⇒ 15 + 1 = 16; T ⇒

20 + 1 = 21; H ⇒ 8 + 1 = 9; E ⇒ 5 + 1 = 6; R ⇒ 18 + 1 =

19.

21. 6 − 4 = 2; 5 − 3 = 2; 2 + 2 = 4.

8 − 6 = 2; 4 − 2 = 2; 2 + 2 = 4

8 − 3 = 5; 7 − 2 = 5; 5 + 5 = 10.

22.

∆FAE; ∆FAI; ∆FIE; ∆CBD; ∆CBJ; ∆CJD; ∆AIJ; ∆BJI; ∆BJA; ∆AIB;

∆IED; ∆JDE; ∆JDI; ∆IEJ; ∆GAB; ∆GAI; ∆GJI; ∆GJB; ∆HJI; ∆HDE;

∆HEI; ∆HJD; ∆AJF; ∆EFJ; ∆BCI; ∆CDI; ∆IBD; ∆JEA; .

23. 24. Delhi is capital of India. But, here India is called Pakistan.

25. Indian teachers who are also advocates can be represented

by the region common to all the three circles. Such region is

marked ‘3’.

26. ‘Economic’ survey of India’ is published every year by

Economic division of Ministry of Finance, Government of

India.

27. The term ‘food security’ implies sufficient supplies of food at

affordable prices.

28. Demand curve is a line on a graph showing the quantity of a

goods consumers would be able and willing to buy in a given

market and specified time period over a range of possible

prices, other things being equal. The demand curve is

depicted as downward-sloping from left to right.

29. The Preamble to the Constitution embodies the objectives

and basic purposes which indicate the fundamental principles

of new order.

30. Andhra Pradesh is the first State in India to have been formed

on a purely linguistic basis. On October 1, 1953, eleven

districts of the Madras State were put together to form a new

Andhra State with Kurnool as capital. On November 1, 1956,

in accordance with the recommendations of the State

Reorganization Commission, the Andhra State was enlarged

and Hyderabad was made the capital.

TARGET NO-1 CLASSES (A UNIT OF) EXECUTIVE MAKERS PVT.LTD. V-157

FOR DOWNLOADING BOOKS : www.executivemakers.com Page-40

31. Nana Sahib, born on May 19, 1824 as Dhondu Pant, was a

rebel Indian leader during the Great Revolt of 1857. As the

adopted son of the exiled Maratha Peshwa Baji Rao II, he

sought to restore the Maratha confederacy and the Peshwa

tradition.

32. The Indo-Soviet Treaty of Friendship and Cooperation was

signed between India and Russia on August 9, 1971 that

specified mutual strategic cooperation. Nikita Khrushchev

was the President of Russia (USSR) in 1971.

33. The Government of Portuguese India was started in 1505,

seven years after the discovery of the sea route to India by

Vasco da Gama with the nomination of the first Governor

Fransciso de Almeida, then settled at Kochi. Alfonso de

Albuquerque succeeded Almeida in 1509. He conquered Goa,

Malacca and Hormuz. He died in Goa in December 1515.

34. The Ganges Delta, formed mainly by the large, sediment

laden flows of the Ganges and Brahmaputra river, is the

World’s largest delta, at about 59,000 sq.km.

35. Moisture Index is a measure of the water balance of an area

in terms of gains from Precipitation (P) and losses from

Potential Evapotranspiration (PE). The Moisture Index (MI) is

calculated thus:

MI= 100 P−PE

PE

Air pressure or Atmospheric pressure is the pressure exerted

by the atmosphere as a result of gravitational attraction

exerted on the column of air lying above a particular point.

Atmospheric pressure, measured in millibars, decreases

logarithmically with height. Absolute humidity is the density

of the water vapour present in a mixture of air and water

vapour, that is , the ratio of the mass of water vapour to the

volume occupied by the mixture., usually measured in grams

per cubic centimetre. Environmental lapre rate is the fall in

temperature of stationary air with height, at the rate of 1°C

per 165 metres.

36. A plan bearing both male and female flowers is called

Monoecious.

37. ‘Jhum’ is a type of cultivation. It is primitive form of

agriculture and is also called ‘slash and burn’ type of

cultivation.

38. Quinine is obtained from the bark of the cinchona tree. It is

also synthesised in the laboratory. Quinine was the first

effective treatment for malaria caused by Plasmodium

falciparum. It is a natural white crystalline alkaloid.

39. Computer memory refers to the physical devices used to

store programs (sequence of instructions) or data.

40. Bronze: An alloy of copper with upto one-third tin.

Brass: A yellow alloy of copper and Zinc.

German silver: A white alloy of nickel, zinc and copper.

Type metal: An alloy of lead, antimony and tin.

41. During night, plant releases carbon dioxide.

42. The calorie requirement of the body increases in winter as

compared to summer because more calories are necessary to

sustain body temperature.

43. Copper sulphate is used as a fungicide. Penicillin is an

antibiotics produced naturally by moulds of the genus

Penicillum. Urea is a nitrogenous compound used as fertiliser.

Malathion is an insecticide containing phosphorus with low

toxicity to other animals.

44. When plant diversity is maintained in natural habitat, the

conservation is called in situ. In situ is a Latin term which

means “in its place,” “in its original place”. In vivo means

“taking place in a living organism”. In vitro means “taking

place in test tube or other laboratory environment.”

45. Lithosphere refers to rigid part of the earth’s crust, that

extends from a depth of about 65 kilometre below the

surface and comprises of the Sial, the Sima and the upper

Mantle. Below it is the softer, weaker layer known as the

Asthenosphere.

46. Adam’s apple is the projection at the front of the neck

formed by the thyroid cartilage of the larynx.

47. GSLV (Geosynchronous Satellite Launch Vehicle) is an

expendable launch system operated by Indian Space

Research Organization (IRSO). It was developed to enable

India to launch its INSAT-type satellites into geostationary

orbit.

51. Required answer = 20 .

52. Number are 6x and 6y where x and y are co-prime,

Product = 36xy , LCM= 6xy

HCF

LCM=

6

6XY=

36

36XY=

1

60.

53. 0.2304+ 0.1764

0.2304− 0.1764=

0.48+0.42

0.48−0.42=

0.9

0.06=

9×10

6= 15.

54. Part of tank filled in 1 hour = 1

2+

1

3−

1

6=

3+2−1

6=

4

6=

2

3. Hence, the tank will be filled in

3

2= 1

1

2 hours.

55. C.P. = Rs. 100, Marked price= 150, SP = 150×60

100= Rs. 90 , Loss = 10% OR Profit/loss percent =

50 − 40 −50×40

100 = −10% Negative sign shows loss.

56. sum of ratios = 2+3+7=12 Required difference = 7−2

12× 48 =

20 kg.

57. Weight of new man = 56 + 8 × 2.5 = 76 kg.

58. 2%=3 , 100%=3

2× 100 = Rs. 150.

59. Expenditure = Rs. 5x, saving = Rs. x , Income = Rs. 6x , New

income= Rs. 7.5x, Savings = Rs. 1.15x, Expenditure = Rs. 6.35x

Percentage increase = 1.35𝑥

5𝑥× 100 = 27%.

60. Distance covered by A in half an hour = 4 km. Relative speed

of B = 10 – 8 =2 kmph, Required time =4

2= 2 hours i.e. 10

a.m.

TARGET NO-1 CLASSES (A UNIT OF) EXECUTIVE MAKERS PVT.LTD. V-157

FOR DOWNLOADING BOOKS : www.executivemakers.com Page-41

61. S.I. in two years =

225×2

3= Rs. 150, R =

150×100

P×2=

7500

P , Difference =

PR2

100 2 ⇒ 3 =

P×7500×7500

P2×100×100⇒ 3P2 = P × 75 × 75 ⇒ P =

75×75

3= Rs. 1875.

62. Let’s take an example: Let length of rectangle = 60 units,

Breadth = 40 units. Perimeter = 2(60+40)=200 units , Area =

60×40=2400 sq.units .Side of Square = 200

4= 50 units.

63.

BAD = 30° , If BD = r units, then AD = h = BD cot 30° =

3r units , AB = 𝑙 =BD

sin 30°= 2𝑟 units

13𝜋𝑟2ℎ

𝜋𝑟𝑙=

1

3⇒

𝑟ℎ

3𝑙 =

1

3⇒ 𝑟ℎ = 𝑙 , 𝑟 × 3𝑟 = 2𝑟 ⇒ 𝑟 =

2

3.

64. (2𝑛1−4

𝑛1× 90° = 160° ⇒ 180𝑛1 − 360 = 160𝑛1

⇒ 20𝑛1 = 360 ⇒ 𝑛1 = 18.

Number of sides = 18+5=23.

65. According to questions 2

3𝜋𝑟3 = 3𝜋𝑟2 ⇒ 𝑟 =

9

2= 4

1

2𝑢𝑛𝑖𝑡𝑠 .

66. 𝑥 = 𝑦 ⇒ 2𝑡 =2𝑡−1

3⇒ 6𝑡 = 2𝑡 − 1 ⇒ 4𝑡 = −1 ⇒ 𝑡 = −

1

4.

67. 𝑥 = 5+1

5−1= 5+1

5−1×

5+1

5+1= 5+1

2

5−1=

5+1

2 ,

𝑥2 + 𝑥 − 1 = ( 5+1

2)2 +

5+1

2− 1 =

5+1+2 5

4+

5+1

2− 1 =

3+ 5

2+

5+1

2− 1 =

3+ 5+ 5+1−2

2= 5 + 1.

68.

y=0 is an equation of x-axis. y=x is the equation of OA.

Equation of AB II y-axis is x=c.

69.

k II 𝑙 II m ⇒ BOA = 45° ⇒ AOB = a° and DOB =

b° so a° + b° = AOB = 45°.

70. SR =

distance between center 2 − r1 − r2 2= 13 2 − 5 2 =

18 × 8 = 12 cm.

71.

AB. AC + AE. DE = AE2

AB.AC = AE(AE-DE) = AE. AD

72. It is obvious from the graph.

73. Increase in profit percent = 65−45

45× 100 = 44%.

74. Range = 70-40 = 30.

75. 40 = income−20000

20000× 100 ⇒ 8000 = income − 20000 ⇒

income = Rs. 28000 crore.

76. Here, my friend is used to speaking …. should be used .

77. Here, has flowed …. should be used.

Look : Fly →Flew(Past) →Flown (Past Participle).

78. Dispense with somebody/Something means: to stop using

somebody/ something because you no longer need them or

it.

79. Be cut out for something / be cut out to be something means:

to have the qualities and abilities needed for something.

81. The word Indiscriminate (Adjective) means: action without

thought; undifferentiated.

TARGET NO-1 CLASSES (A UNIT OF) EXECUTIVE MAKERS PVT.LTD. V-157

FOR DOWNLOADING BOOKS : www.executivemakers.com Page-42